You are on page 1of 66

PROMETRIC

PREPERATION
FOR DENTIST

1|Page
01. Which one of the following is NOT a purpose of dentistry?
A Improve a patient's ability to eat
B Improve a patient's ability to talk
C Improve the appearance of their face
D Straighten a patient's teeth
The correct answer is C

02. Which is the key principle of the practice of dentist? (ethics)


A Relief of dental pain
B Earn a living
C Provide cosmetic dentistry to patients
D Provide botox and collagen to patients
The correct answer is A

03. Which one of the following statements is NOT a principle of the GDC?
A Putting the patient's interest first
B Respecting the patient's dignity
C Charging for dental treatment
D Protecting the patient's confidential information
E Being trustworthy
The correct answer is C

04. Which is a rare source of pain felt in the mouth?


A The tooth pulp
B The gums
C The bone
D The tongue
E The heart
The correct answer is E

05. Which is NOT a dental problem?


A Tooth decay
B Painful gums
C Aching jaw joint
D Pain on the face
E Sore or painful tongue
The correct answer is D

06. What is NOT part of the normal surgery equipment?


A A delivery unit
B An X-ray set
C An electosurgery unit
D Suction equipment
The correct answer is C

07. What is the key objective of an operating position?


A To gain good access to the operating area
B To gain good visual access to the operating area and protect the dentist's back
C To keep the patient comfortable
D To allow the nurse to see the treatment
The correct answer is B

08. When would a dentist normally stand to operate on a patient?

2|Page
A When they feel like it
B When the patient cannot lie flat on the chair
C When an upper molar tooth is being extracted
D When doing treatment on lower molar teeth
The correct answer is C

09. What are the key principles of correct body posture for a dentist when operating?
A To protect the dentist's back
B To protect the dentist's neck
C To protect both neck and back from injury
D To maximise comfort for the patient
The correct answer is C

10. A patient allows you to take series of digital photographs of their teeth. In which one of the following situations is a
further specific permission from the patient NOT then required for subsequent use of these same images?
A Lending the images to a professional colleague for them to use in a lecture they are due to give in the patient's home
town
B Lending the images to a professional colleague for them to use in a lecture they are due to give at an international
conference being held overseas
C Allowing access to these images by a dental hygienist who sees the patient within your own practice
D Re-use of one or more of the images on your practice website
E Use of the image in a scientific article to be published within a professional journal
The correct answer is C

11. A person claiming to be acting on behalf of one of your patients contacts you out of the blue and requests
confirmation of all the dates and times of appointments attended by the patient over the past 18 months. All but
one of the following steps might be regarded as prudent and necessary safeguards before providing the requested
information. Which one is NOT an appropriate initial response?
A Suggest to the person that it might be more helpful if the patient in question were to contact you in person (without
disclosing whether or not the person is / was indeed a patient)
B A signed and dated authority from the patient to release the information to the named person
C Ask the person to explain the nature of their involvement in the matter and their relationship to the patient
D Ask the person to explain the nature and purpose of the request and how the information might be used
E Ask the person to seek a Court Order compelling you to disclose the requested information
The correct answer is E

12. All but one of the following parties can demand access to your records relating to a particular patient. Which one
cannot do so ?
A The patient
B A solicitor acting on behalf of the patient, upon supplying a signed authority from the patient to release copies of the
records
C An officer representing the General Dental Council, upon supplying a signed authority from the patient to release
copies of the records
D A police officer (in the absence of the patient's agreement, or some form of legal authority to have access to or seize
the records)
E A coroner
The correct answer is D

12. Which one of the following statements is incorrect ?


A A ‘contemporaneous’ record is one that is created at the time of the event(s) to which the record relates
B It is important to record details of any treatment and/or referrals recommended but declined by the patient

3|Page
C It is just as important to record discussions, explanations and conversations that you have with a patient as it is to
record details of the actual procedures carried out
D Laboratory instruction sheets have no continuing value as part of the clinical record, once the work has been fitted
E Clinical records should contain a dated entry demonstrating each occasion that the patient's medical history, current
medication etc. is checked and/or updated
The correct answer is D

13. A patient can be considered to be ‘competent' to provide (or withhold) consent on their own behalf in relation to a
treatment procedure proposed for them, if certain conditions are satisfied. Which of the following is NOT one of
them?
A They have had a similar procedure carried out in the past, so it is reasonable to assume that they would have no
objection to the procedure now being considered
B They must be able to understand what is being proposed for them, and why
C They must be able to consider and weigh up their options and ask appropriate questions (if necessary) before
reaching a decision
D They must be able to communicate their decision clearly
E They are aged 16 years or over (based on the presumption of ‘capacity' unless it can be shown otherwise)
The correct answer is A

14. The information that you provide to a patient about treatment proposed for them needs to include four of the five
elements listed below. Which ONE is the exception?
A Why the treatment is being proposed for them
B What the procedure actually involves and what the effect is likely to be
C A detailed review of the relevant evidence base, providing the patient with a list of references to relevant peer-
reviewed scientific articles
D A balanced explanation of any risks and limitations, as well as any potential benefits of the treatment under
consideration
E A summary of any alternative treatment options (including that of doing nothing), and how they compare
The correct answer is C

15. Which of the following is NOT a component of interpersonal communication?


A Verbal communication
B Hypnosis
C Paralinguistic communication
D Non-verbal communication
The correct answer is B

16. Which of the following are key communication skills in work with patients?
A Active listening
B Suggestion
C Story telling
D Seeking confirmation
The correct answer is A

17. Steam sterilization of dental instruments is accomplished at:


A 134°C for 12 min
B 134°C for 3 min
C 100°C for 3 min
D 100°C for 30 min
The correct answer is B

18. Which one of the following cannot be safely reprocessed?


A A flat plastic instrument

4|Page
B A pair of extraction forceps
C A hypodermic needle
D A sickle scaler
The correct answer is C

19. Which one of the following vaccinations is essential for clinical dental staff?
A Hepatitis A
B Hepatitis B
C Hepatitis C
D Hepatitis D
The correct answer is B

20. Which one of the following is NOT a characteristic associated with irreversible pulpitis?
A The tooth is tender to percussion
B The pain is dull
C Heat can elicit the pain
D The pain may be relieved by analgesics
E The pain lasts for minutes after the stimulus has been removed
The correct answer is A

21. Which one of the following is most characteristic of the pain associated with trigeminal neuralgia?
A The pain lasts for hours
B The pain may be initiated by ‘trigger' points
C The pain may be relieved by paracetamol
D The pain usually keeps the patient awake at night
E The pain is dull and throbbing
The correct answer is B

22. Which one of the following relates to advice on decontamination in primary care dental practices?
A HTM 01-01
B HTM 01-02
C HTM 01-03
D HTM 01-04
E HTM 01-05
The correct answer is E

23. If a patient has suffered a myocardial infarction, for how long is it usually advised to delay elective dental treatment?
A 1 month
B 3 months
C 6 months
D 9 months
E 12 months
The correct answer is C

24 . As a right-handed dentist, ideally where should your dental nurse be whilst assisting?
A On the same side as the dentist, standing up
B Behind the dentist
C On the left-hand side, sitting down, facing towards the patient
D On the same side as the dentist, sitting down
E On the left-hand side standing up, facing towards the patient
The correct answer is C

25 . When should PPE be worn by the dentist?

5|Page
A Whilst collecting the patient from the waiting room
B For taking medical histories
C Whilst examining or treating the patient
D Only when the nurse is wearing PPE
E At all times
The correct answer is C

26. What is the objective of vitality testing a tooth?


A To establish the vitality of a tooth's nerve and blood supply
B To determine if the tooth is still alive
C To establish the vitality of a tooth's nerve
D To establish the vitality of a tooth's blood supply
E To determine if the tooth requires root canal therapy (RCT)
The correct answer is A

27. Which one of the following is not a form of sensibility testing in dentistry?
A Coagulation screen
B Transillumination
C Pulse oximetry
D Laser Dopler flowmetry (LDF)
E Transmitted laser light
The correct answer is A

28. Three of the following are potential clinical implications from percussion testing a tooth. Select the three correct
options.
A Tooth is extruded
B Presence of apical or lateral periodontitis
C Presence of a pulpitis
D Tooth is non-vital
E Tooth has an existing root filling
The correct answers are A, B, C

29. Which one of the following is NOT a valid reason for the absence of a clinical response when conducting vitality
testing?
A Reversible pulpitis
B Pulpal necrosis
C Pulp absence
D Root canal therapy (RCT)
The correct answer is A

30. Which two of the following are clinical reasons for obtaining a false positive when performing an electrical pulp test
(EPT)?
A Pulp chamber full of pus
B Multirooted tooth with both vital and non-vital roots
C Nerve damage, however, blood supply intact
D Large amount of secondary dentine
E Large restoration
The correct answers are A, B

31. Which one of the following defines Quality Assurance Rating 2?


A Errors of patient preparation, exposure, positioning, processing or film handling, which render the radiograph
diagnostically unacceptable

6|Page
B No errors of patient positioning, exposure, positioning, processing or film handling
C Some errors of patient preparation, exposure, positioning, processing or film handling, but which do not detract from
the diagnostic utility of the radiographs
The correct answer is C

32. Which of the following are required for reading radiographs?


A Knowledge of ‘normal' anatomy
B A systematic and logical way of reading radiographs
C Digital X-ray system
The correct answers are A, B

33. When labelling radiographs, which one of these types of information does NOT need to be included?
A Patients name
B Dentists name or initials
C Patient D.O.B
D Dentist D.O.B
E Date radiograph taken
The correct answer is D

34. Which of the following is a useful mnemonic for a surgical sieve?


A Vitamin E
B Vitamin C
C Vitamin K
D Metabolic
E Clinical
Answer: B. The mnemonic vitamin C is commonly used, which stands for: Vascular, Infection, Trauma /toxicity,
Autoimmune, Metabolic, Idiopathic, Neoplastic and Congenital
35. Which of the following is NOT one of the headings normally considered as part of the surgical sieve?
A Neoplasia
B Infection
C Traumatic
D Vascular
E Thrombotic
The correct answer is E

36. In terms of special investigations, which of these is NOT a routine investigation for a dental complaint?
A Intraoral X-ray
B Extraoral X-ray
C MRI
D Vitality testing with heat
E Vitality testing with cold
Answer: C. MRI is not a routine dental investigation and normally would only be considered if there is suspicion of
intracranial pathology

37. You are working your first week in Foundation Training in a Maxillofacial Surgery Department. You are asked to see
a 46-year-old man who smokes 30 cigarettes per day. He has been referred by his General Dental Practitioner. The
referral letter indicates concerns about papules in the area of the hard palate and gives a provisional diagnosis of
nicotine stomatitis. What is a papule?
A Circumscribed palpable elevation, less than 5 mm in diameter
B Elevated area, more than 5 mm in diameter
C Small visible fluid accumulation in epithelium, less than 5 mm in diameter
D Solid mass under or within the mucosa, more than 5 mm in diameter
E Visible accumulation of pus in epithelium

7|Page
The correct answer is A

38. In the FDI system for tooth identification what tooth is indicated by the number 54?
A Upper left first permanent premolar
B Upper right first deciduous molar
C Upper right second permanent molar
D Lower left deciduous canine
E Lower left permanent canine
The correct answer is B

39. When carrying out a BPE examination of an upper right sextant for a fully dentate patient, the deepest probing
depth was with the black band partially obscured, furcation involvement was identified. What is the correct BPE score
for the sextant?
A 2
B 3
C 3*
D 4
E 4*
The correct answer is C

40. Clinical attachment loss (CAL) at a particular site is determined by which one of the following?
A Adding the measurement of recession (millimetres) to the pocket depth (millimetres)
B Measuring the pocket depth only (millimetres)
C Adding the measurement of recession (millimetres) to the BPE code for the sextant
D Measuring the distance from the cement–enamel junction (CEJ) to the mucogingival junction (millimetres)
E Adding the pocket depth (millimetres) to the BPE code for the sextant
The correct answer is A

41. When considering whether to re-root treat an upper first permanent molar, which has previously been root treated
and the root treatment has failed, which one of the following is irrelevant to your decision?
A The probability of success of re-root treatments in dental hospital patients
B The probability of success of molar apicectomies
C The likelihood of pain if the root treatment fails
D The threat of litigation you faced from the last patient on who you performed a root treatment which failed
E The patient's age
The correct answer is D

42. When considering the best treatment plan for a patient complaining of an acutely sensitive UL6, which of the
following is the LEAST preferred possible option?
A Doing nothing
B Placing a sedative dressing
C Placing a steroid antibiotic dressing
D Commencing a root treatment
E Crowning the tooth
The correct answer is C

43. How are the risks and probabilities of success from treatment best assured?
A Practice audit of success rates
B Individual audit of success
C Memory
D Systematic review of randomized controlled trials in dental hospitals
E What you learnt at university
The correct answer is B

8|Page
44. The order in which a primary survey should be carried out for a patient who you find collapsed in your surgery
driveway is:
A Airway, circulation, exposure, breathing, neurological disability
B Breathing circulation, drugs and neurological disability, exposure and airway
C Airway, exposure, drugs and disability, breathing and circulation
D Airway, breathing, circulation, drugs and neurological disability, exposure
E Circulation, breathing, airway, exposure and drugs and neurological disability
The correct answer is D

45. An adult patient in your waiting room collapses, and your examination reveals that they have stopped breathing and
are non-responsive. What ideally should you do next?
A Check for danger, airway, breathing and circulation and call an ambulance
B Call an ambulance and return with an AED machine and commence CPR with 30 chest compressions at a rate of 100–
120 with two rescue breaths whilst attaching the AED machine
C Call an ambulance and return with an AED machine and commence CPR with 15 chest compressions at a rate of 100–
120 with two rescue breaths
D Call an ambulance and attach the AED machine and give a shock as instructed
E Try and rouse the patient, check for ABCDE and measure the blood glucose.
The correct answer is B

46. The cardiac rhythms most likely to be result in a cardiac arrest are:
A Ventricular tachycardia, atrial fibrillation, PEA
B Sinus tachycardia, atrial fibrillation, PEA
C Ventricular fibrillation, atrial fibrillation, PEA
D Ventricular fibrillation, heart block and PEA
E Ventricular fibrillation, asystole, PEA
The correct answer is E

47. A patient complains of central chest pain during a dental extraction. The pain is like a heavy weight in nature and he
complains that it goes down his left arm. He has become pale and is sweating a lot. How should this be managed in the
first instance?
A Call an ambulance and monitor the patient
B Monitor the patient and provide oxygen (15 L/min)and aspirin
C Lie patient flat, provide oxygen(15 L/min) and begin chest compressions
D Sit the patient up, provide oxygen (15 L/min), two puffs of GTN spray and wait for no longer than 5 minutes before
reassessing
E Sit the patient up and provide oxygen (15 L/min), aspirin and call an ambulance
The correct answer is D

48. The key diagnostic features of an acute severe asthmatic attack include:
A Wheezing, increased respiratory rate, bradycardia
B Wheezing, increased respiratory rate, tachycardia
C Cyanosis, bradycardia, aggression
D Increased respiratory rate, bradycardia, cyanosis
E Inability to talk, reduced respiratory rate, bradycardia
The correct answer is B

49. If a patient fails to respond to two to four puffs of salbutamol and their asthma attack deteriorates further, what
would be your next series of actions?
A Call an ambulance, administer up to 10 puffs of salbutamol via a spacer and monitor the patient
B Administer 10 puffs of salbutamol via a spacer and provide 15 L/min of oxygen
C Provide 15 L/min oxygen via a face mask and call an ambulance

9|Page
D Administer 1000 mg salbutamol via a nebuliser
E Administer 200 mg hydrocortisone IM, 1000 mg salbutamol via a spacer and call an ambulance
Sorry, you have selected the wrong answer.
The correct answer is A

50. A patient undergoes an inferior dental block, and complains of tightness of the chest, wheezing, difficulty in
swallowing, itching and a swollen tongue. What would you do in response to this event?
A Lie patient head down, provide oxygen (15 L/min) and administer 0.5 mL of 1 : 1000 IM epinephrine (adrenaline)
B Sit the patient up, provide oxygen (15 L/min) and administer 0.5 mL of 1 : 1000 IM epinephrine (adrenaline)
C Call an ambulance, provide oxygen (15 L/min) and monitor the patient
D Call an ambulance and administer IM hydrocortisone, IM chlorphenamine and provide oxygen (15 L/min)
E Call an ambulance, monitor the vital signs, provide oxygen (15 L/min) and administer 0.5 mL of 1 : 1000 epinephrine
(adrenaline) IM
The correct answer is E

51. With regards to the above scenario, what are the cellular events that trigger the reaction?
A An antigen is presented by a macrophage to a B lymphocyte cell, resulting in the release of a large quantity of
antibodies
B An antigen is recognised by a mast cell membrane receptor (IgE), which results in the release of a series of mediators
or acute inflammation from intracellular granules
C An antigen activates the classical pathway of complement resulting in the release of series of anaphylatoxins
D An antigen activates the proliferation of T helper cells, which results in the activation of B cell
E An antigen is phagocytosed by polymorphoneutrophils (PMN) which results in the release of a number of acute
inflammatory mediators
The correct answer is B

52. A type 1 diabetic patient attends for an extraction, but you keep her waiting for 45 minutes, and whens she sits in
the chair she complains to you of feeling light headed and a little faint and tells you she suspects a ‘hypo' episode is
imminent. What would you do for this patient in the first instance?
A Provide 15 L/min oxygen and call an ambulance
B Administer 1 mg of glucagon IM
C Administer 25 g Glucogel into buccal mucosa
D Provide a glucose drink with at least 10 g glucose and repeat 10 minutes later
E Lie patient down in the head down position and administer 15 L/min oxygen
The correct answer is D

53. What are recognised signs of a hypoglycaemic event?


A Central chest pain
B Swelling of the oral mucosal surfaces
C Confusion and agitation
D Shortness of breath
E An itchy rash
The correct answer is C

54. Fluoride is effective in caries control because:


A It is easily digested
B It favours remineralization over demineralization in the caries process
C It stimulates saliva
D All patients receive exposure to fluoride through public water supplies
The correct answer is B

55. A caries risk assessment:


A Is necessary only for patients with active carious lesions

10 | P a g e
B Is for children only
C Is the responsibility of a dental care professional
D Is a key part of every patient assessment
The correct answer is D

56. Risk factors for dental caries:


A Are behaviourally determined
B Are easily reduced by the dental team
C Remain the same throughout life
D Are principally related to diet
The correct answer is A
57. Acute fluoride toxicity can occur when:
A The patient lives in an area of water fluoridation and additionally uses a fluoride mouth rinse
B At fluoride doses in excess of 1.0 mg/kg
C At fluoride doses in excess of 5.0 mg/kg
D The patient's caries risk assessment is low
The correct answer is B

58. Which of the following is not a feature of a good manual toothbrush for routine use?
A Handle size appropriate to user age and dexterity
B Head size appropriate to the size of the patients mouth
C Use of firm bristles
D Use of round-ended nylon or polyester filaments 0.23mmin diameter
E Use of bristle pattern that enhances plaque removal in the approximal surfaces
The correct answer is C

59. Proximal Bleeding Scores in patients who complied with long-term maintenance were:
A <5%
B 15%
C 25%
D 35%
The correct answer is A

60. Which one of the following is NOT a patient factor that guides oral hygiene advice?
A Age of subject
B Susceptibility of subject to gingivitis and periodontitis
C Social class
D Past periodontal disease experience
E Manual dexterity
F Knowledge and motivation in oral hygiene practices
The correct answers is C

61. The twice-daily use of chlorhexidine gluconate has been shown to reduce plaque by:
A 19–56%
B 20–36%
C 45–61%
D 20–23%
E 20–32%
The correct answer is C

62. Which of the following activities is not part of needs-based professionally performed supportive care?
A Monitoring of plaque levels
B Plaque removal

11 | P a g e
C Targeted oral hygiene
D Monitoring of clinical attachment level
E Subgingival irrigation
The correct answer is E

63. Smoking increases the risk of periodontitis by a factor of:


A 1–2
B 2–2.95
C 3.25–7.28
D 7.5–9.5
The correct answer is C

64. Which of the following factors are not associated with periodontitis?
A Diabetes mellitus
B Rheumatoid Arthritis
C Stress
D Higher social class
E Obesity
The correct answer is D

65. The two most common age groups for dental trauma are:
A 0–1 and 7–10 years
B 2–4 and 6–7 years
C 3–5 and 12–15 years
D 2–4 and 7–10 years
The correct answer is D

66. The most common cause of dental trauma in preschool children is related to:
A Falling
B Cycling
C Road traffic accident
D Assault
The correct answer is A

67. The role of the dental team in preventing dental trauma does NOT include:
A Raising concerns of suspicion of neglect and non-accidental injury
B Providing mouthguards
C Providing advice to patients, schools and clubs regarding prevention and emergency treatment
D Advising patients to avoid contact sports and cycling
The correct answer is D

68. The first permanent molar:


A Replaces the last deciduous molar
B Is complete at age 7 years
C Erupts after the central incisors have just become visible in the mouth
D Begins to calcify around the time of birth
E Is unimportant as it can be replaced by the second permanent molar
The correct answer is B

69. The order of eruption of the deciduous dentition is:


A Central incisors, lateral incisors, first molars, canines, second molar
B First molars, central incisors, lateral incisors, second molars, canines

12 | P a g e
C Central incisors, lateral incisors, canines, first molars, second molars
D Central incisors, lateral incisors, first molars, second molars, canines
E Central incisors, first molars, lateral incisors, canines, second molars
The correct answer is A

70 . The first deciduous molar


A Erupts at the age of 2.5 years
B Begins calcifying at birth
C Is the third tooth of the deciduous dentition to erupt
D Erupts roughly 6 months before the second deciduous molar
E Erupts at the same time as the deciduous incisors
The correct answer is C

71 . In the secondary dentition, which of the following is NOT true?


A The first molars and incisor erupt around the same time
B The root of the canine is complete around the age of 15 years
C The roots of the first permanent molars are complete around the age of 9 years
D The beginning of calcification of the maxillary lateral incisors is around 1 year of age
E The maxillary canines erupt around 10 years of age
Answer: E. Upper canines erupt at 12 years, lowers (mandibular) at age 10 years.

72. Hypodontia is:


A Six or more congenitally missing teeth
B A complete absence of teeth
C Any number of missing teeth
D Up to six excess teeth
E Absence of supernumerary teeth
The correct answer is C

73. Supernumerary teeth:


A Refers only to teeth that resemble those of the normal series
B Are synonymous with supplemental teeth
C May include mesiodens
D Are always tuberculate or barrel-shaped
E Are always conical or peg-shaped
The correct answer is C

74. Hyperdontia is associated with which one of the following syndromes and conditions?
A Ankylosing spondylitis
B Cleidocranial dysostosis
C Cherubism
D Epidermolysis bullosa
E Rickets
The correct answer is B

75. Germinated teeth:


A Are more common in the deciduous than the permanent dentition
B Comprise 0.1–4.5% of permanent teeth
C Comprise 0.1–0.8% of the deciduous teeth
D Have separate pulp chambers
E Are most common in English-speaking nations
The correct answer is A
76. Dilacerated teeth:

13 | P a g e
A Are most commonly caused by trauma to the primary predecessor tooth
B Arise subsequent to 10% of trauma to primary teeth
C Cannot be managed by surgical exposure
D Should be extracted
E Are teeth with a sharp bend towards the apex of the tooth
The correct answer is A

77. Dens evaginatus:


A Can be the cause of dental pain
B Are usual unilateral
C Occur more frequently in the maxillary teeth than the mandibular
D Contain pulp tissue in the majority of cases
E Are more frequently seen in people of Mongoloid origin
The correct answer is E

78. The defects of dentine and enamel index:


A Has eight main categories (excluding ‘other')
B Can only be used for screening surveys
C Was recommended by the WHO for use in oral health surveys in 1967
D Is based on a etiological criteria
Sorry, you have selected the wrong answer.
The correct answer is B

79. Dentinogenesis imperfect


A Is an autosomal recessive genetic defect
B Is a separate condition to osteogenesis imperfecta
C Causes the teeth to have large pulps due to lack of dentine production
D Is also known as dentine dysplasia
E On radiographs typically shows a shortened root
The correct answer is E

80. Fluorosis:
A Is due to fluoridation of the water in 70% of cases
B Is of public health significance if the Community Fluorosis Index is below 0.4
C Has a Dean's Index Score of 1 if there are small white flecks or spots 1–2 mm in diameter
D Has a Dean's Index Score of 1 if there are small white areas covering up to 25% of the teeth surface
E Is severe if all of the tooth surfaces are involved and if minute pitting is present and brown staining is common
F Intensity of fluorosis depends only on the age of the child when they are exposed to the excess fluoride
The correct answer is C

81 . Mild fluorosis can be distinguished from non-fluoride opacities because:


A Fluorosis affects the whole crown
B Fluorosis lesions are often round or oval
C Fluorosis lesion shade off imperceptibility into the surrounding normal enamel
D Fluorosis occurs on single teeth
E Mildly fluorosed teeth may seem etched and rough to an explorer
The correct answer is C

82. Deciduous central incisors:


A Begin to calcify at 3–4 months of age
B Crowns are completed at around 2 months of age
C Erupt at about 13 months of age
D Have fully formed root by the age of 5 years

14 | P a g e
E Are the second teeth to erupt
The correct answer is B

83. Permanent maxillary canines:


A Begin to calcify at 4–5 months of age
B Crowns are completed at 4–5 years of age
C Erupt at around 9–10 years of age
D Complete their roots at around 15 years of age
E Are the last permanent teeth to erupt
The correct answer is A

84. The most common eruption sequence of the permanent dentition is:
A
B
C
D
E
The correct answer is B

85. The biofilm covering teeth:


A Is formed from the by-products of bacteria
B May be 5 &mgr;m thick after a period of 24 hours
C Is initially colonised by actinomyces bacteria
D Is initially colonised by Streptococcus sanguinis and S. mitis
The correct answer is D

86. You notice that a patient's 10-year-old amalgam restorations are at a raised level when compared with the
surrounding enamel. Which one of the following would be the most likely diagnosis?
A Amelogenesis imperfecta
B Bruxism
C Creep of the amalgam
D Dental erosion
E Tooth brush abrasion
The correct answer is D

87. A 13-year-old female presents with tooth surface loss and dental sensitivity. Which one of the following would be
your first-line management in such a case?
A Indirect composite onlays
B Direct composite restorations
C Monitor tooth surface loss and sensitivity in 6 months
D Investigate possible causes and instigate preventative measures
E Vital nightguard teeth whitening
The correct answer is D

88. Which one of the following is the most likely cause of the tooth surface loss in a 23-year-old known anorexic?
A Hypomineralised enamel
B Abrasion
C Attrition
D Caries
E Erosion
The correct answer is E

89. Functional stimulation as occurs during eating results in keratinisation of the:

15 | P a g e
A Buccal mucosa
B Floor of the mouth
C Oral gingival epithelium
D Sulcular gingival epithelium
E Junctional epithelium
The correct answer is C

90 . The sulcular epithelium is:


A Stratified squamous keratinised
B Stratified squamous non-keratinised
C Attached to the enamel
D Attached to the cementum
E Limited apically by the cement–enamel junction
The correct answer is B

91. The junctional epithelium in clinically healthy gingival tissue is:


A Stratified squamous keratinised
B 15–20 epithelial cells thick
C Traversed by polymorphonuclear leukocytes
D Attached to the cementum
E Continuous with the oral epithelium
The correct answer is C

92. Increased loading on the teeth:


A Is always associated with tooth mobility
B Has no effect on the orientation of the fibres in the PDL
C Results in a narrowing of the periodontal ligament
D Results in a widening of the periodontal ligament
E Results in loss of collagen fibre attachment to cementum
The correct answer is D

93. Which one of the following is essential for the development of a mature supragingival plaque biofilm?
A Toothbrushing only performed once per day
B Bacteria that can adhere to pellicle
C Incorporation of Fusobacterium nucleatum
D An anaerobic environment within the biofilm
E Frequent sugar intake
The correct answer is B

94. The bacterial species Porphyromonas gingivalis:


A If present in plaque is always associated with periodontal disease
B Is a Gram-positive anaerobe
C Needs to be present in high quantities to cause periodontal disease
D Is an early coloniser in plaque biofilm formation
E Has a low capacity to cause inflammatory changes
The correct answer is E

95. Which one of the following is most correct in relation to chronic periodontitis?
A It usually affects the palatal surfaces of the upper anterior teeth
B The severity is classified by measurements of pocket depth
C It is defined as loss of alveolar bone
D Smoking is the major environmental risk factor
E It is always associated with inflammation of the gingival margin

16 | P a g e
The correct answer is D

96. Which of the following statements is most correct? Acute ulcerative gingivitis is a condition that:
A Occurs in those with pre-existing chronic periodontitis
B Is cured by treatment with a course of antibiotics
C Exhibits necrosis of the interdental papillae
D Is not associated with stress
E Is the result of long-term poor plaque control
The correct answer is C

97. Which one of the following does NOT have a direct effect on the colour of teeth?
A Translucency of the enamel
B Thickness of the enamel
C Degree of mineralisation of the enamel
D Size of the pulp horns
E Colour of the underlying dentine
Answer: D. When the size of the pulp is reduced, it is replaced by dentine; therefore there is an indirect effect, not
direct.

98. Which statement about the properties of enamel is incorrect?


A Enamel is a permeable material
B Enamel is differentially soluble to weak acids
C Enamel can wear down when opposed by porcelain
D Enamel is mostly composed of inorganic hydroxyapatite
E The thickest layer of enamel can be found over the cusp tips
Answer: A. Enamel is a semipermeable material not a permeable material.

99. Which one of the followings is formed by odontoblast-like cells of the pulp?
A Primary dentine
B Secondary dentine
C Reactionary dentine
D Reparative dentine
E Inorganic dentine
Answer: D. This type of dentine is formed when odontoblasts die but pulp retains its vitality and the cavity is very close
to the pulp.
100 . Activation of the C fibres within the pulp results in what type of pain for the patient?
A Sharp and localised
B Sharp and diffused
C Burning and localised
D Burning and diffused
E Pulsating and localised
The correct answer is D

101. A standard UK cartridge of lidocaine with epinephrine contains:


A 4.4 mg of lidocaine and 2.75 &mgr of epinephrine
B 44 mg of lidocaine and 2.75 &mgr of epinephrine
C 44 mg of lidocaine and 27.5 &mgr of epinephrine
D 88 mg of lidocaine and 2.75 &mgr of epinephrine
E 88 mg of lidocaine and 27.5 &mgr of epinephrine
The correct answer is C

102. Which maximum dose of local anaesthetic is correct?


A Articaine 5.1 mg/kg body weight

17 | P a g e
B Lidocaine 4.4 mg/kg body weight
C Lidocaine 8.4 mg/kg body weight
D Mepivacaine 6.6 mg/kg body weight
E Prilocaine 3.8 mg/kg body weight
The correct answer is B

103 . Which one of the following statements about vasoconstrictors in local anaesthetic is true?
A Epinephrine dilates the coronary arteries
B Felypressin dilates the coronary arteries
C Vasoconstrictors shorten the duration of local anaesthesia
D Vasoconstrictor containing local anaesthetics have a higher pH than plain solutions
E Higher concentration of vasoconstrictors are associated with a lower incidence of side effects
The correct answer is A

104 . Which statement on the clinical technique for achieving local anaesthesia for dental treatment is true?
A The most appropriate technique to anaesthetise a lower molar is a buccal infiltration with mepivacaine without
vasoconstrictor
B Palatal soft tissue anaesthesia is required for the provision of plastic restorations in upper molar teeth
C An inferior alveolar nerve block is a more successful technique than a buccal infiltration for producing anaesthesia of
the lower lateral incisor
D A buccal infiltration distal to the apex of the upper second premolar tooth is the most appropriate way to produce
analgesia for tooth restoration
E A buccal infiltration mesial to the apex of the mesiobuccall root of an upper first molar tooth is the most appropriate
way to produce analgesia for tooth restoration
The correct answer is D

105. You have placed a butterfly clamp on the upper right lateral incisor and are about to start an endodontic
treatment. Which one of the following statements is correct?
A If you drop a file, the rubber dam will obscure your vision and you will not be able to retrieve it
B The patient will find the rubber dam uncomfortable as the water and fluids will start pooling in the mouth
C The butterfly wings will reduce your access to the palatal surface of the tooth
D You will not be able to prepare a class IV cavity for a composite restoration while the clamp is on
E The hypochlorite wash can dissolve the dam
The correct answer is D

106. Which one of these tooth isolation techniques CANNOT be used when taking the final impression of a prepared
crown?
A High-volume suction
B Rubber dam
C Saliva ejector
D Svedopter (flange)
E Cellulose pad
The correct answer is B

107. Which one of the following CANNOT be used to invert the dam once in place?
A A high volume of air
B BPE probe
C No. 1-2 plastic instrument
D A high volume of water spray
E A No. 23 explorer
The correct answer is D

108 . Which one of the following statements about cavity preparations is FALSE?

18 | P a g e
A Class II cavity preparations for amalgam restorations must be extended onto the occlusal surface to provide adequate
retention
B If thin walls remain following a multisurface cavity preparation, these should be reduced and a cuspal coverage
restoration provided
C Resin composite should not be used to restore a cavity with subgingival margins
D The ‘slow’ hand piece should be used with water
E Parallel-sided fissure burs will create a cavosurface angle that is greater than 90°
The correct answer is A

109. Which one of the following protocols should be followed when placing a direct restoration in a functional tooth?
A Check the occlusal contacts on the restoration after placement
B Check the occlusal contacts before cavity preparation
C Check the occlusal contacts before cavity preparation and after restoration placement
D Check the occlusal contacts before cavity preparation. If occlusal contacts exist at the junction of the cavity, then
extend the preparation to allow occlusal contacts entirely on the restoration. Check the occlusal contacts following
restoration placement
E Check the occlusal contacts at the following appointment
The correct answer is D

110 . Which of the following mechanisms is the least likely cause of pulpal damage during the cavity preparation?
A Compromising the vascular supply of the pulp by the vasoconstrictor in the local anaesthetic agent
B Compromising the vascular supply of the pulp by using local anaesthetic agents with vasoconstrictors
C Using a rotary instrument with excessive pressure
D Desiccation of the tooth tissue during the etching process
E Bacterial contamination prior to placement of the restoration
The correct answer is B. Using a rotary instrument with excessive water coolant

111 . In which clinical situation should a sealer be used following cavity preparation?
A When 0.5 mm of dentine overlies the pulp
B When 1 mm of dentine overlies the pulp
C When a pulpal exposure has occurred
D Whenever dentine is exposed following cavity preparation
E When the preparation is entirely within enamel
The correct answer is D

112. The appropriate time to restore a tooth is:


A When the tooth becomes sensitive to hot and cold stimuli
B When remineralisation and repair can no longer be expected
C When caries has breached the enamel–dentine junction
D When the tooth begins to look aesthetically displeasing
Answer: B. Caries should be arrested whenever possible and the restorative cycle should only be commenced once the
caries is irreversible.

113 . When preparing a cavity the size of the access is dictated by:
A The type of bur used in the air rotor
B The need to remove excess tooth tissue
C The need to allow access to the decay
D The rule of extension for prevention
Answer: C. Cavity preparation should avoid unnecessary removal of sound tissue and should be as small as possible
whilst allowing access to the decay.

114 . Caries removal should:


A Remove all infected and affected dentine

19 | P a g e
B Remove only affected dentine
C Remove infected dentine
D Be limited to the enamel
Answer: C. Infected dentine must be removed, whilst dentine which is demineralised but not infected can be sometimes
be left in situ, depending on the restorative material used.

115. Caries-affected dentine:


A Contains bacteria
B Is demineralised
C Is the same as caries-infected dentine
D Should always be removed when a cavity is present
Answer: B. Caries-affected dentine is demineralized but without infection and can be left during cavity preparation,
depending on the choice of restorative material.

116 . Selection of the best restorative material does NOT depend on:
A The tooth concerned
B The extent of tooth destruction
C The caries risk status of the individual
D Moisture control
E The order in which the restorations are placed
Answer: E. Restoration placement order should be dictated by circumstances.

117. Glass ionomer cements are most suitable for:


A Restorations of the incisal edge
B Large MOD cavities in posterior teeth
C Patients with a high caries risk
D Definitive posterior restorations
Answer: C. GIC is best for small class III and V cavities and provision restorations, particularly in patients exhibiting high
caries risk.

118 . Good moisture control:


A Is less important using adhesive technique than with amalgam
B Allows a protein film to form on the surface of the tooth
C Is crucial for the placement of both GICs and composites/compounders
D Reduces retention of restorations
E Decreases the risks of marginal leakage
The correct answer is E

119 . Dental amalgam:


A Is not useful for large multisurface restorations
B Shows good adhesive properties
C Is retained in a cavity by mechanical retention
D Is no less technique sensitive than composite restoration materials
The correct answer is C

120. Matrix techniques:


A Are only required for aesthetically appealing restorations
B Are the same no matter what restorative material is being used
C Are required when there has been a small amount of destruction of tooth tissue
D Are used to restore the form and contact relationship of a tooth
Answer: D. A matrix is used to define the periphery of a restoration.

20 | P a g e
121 . Bonding a restoration to a tooth:
A Is only possible with GIC restorations
B Is only possible with composite/component restorations
C Is only possible with amalgam restorations
D Is possible with amalgam, composite and GIC restorations
Answer: D. Bonding is achievable with all three of the major groups of restorative materials.

122 . Bonding amalgam:


A Depends on agents that bond metal oxides to enamel/dentine
B Always involves the use of a resin-based technique
C Is a process that is not technique sensitive
D Is not affected by contamination by saliva
Answer: A. Any adhesive that will bond to both enamel/dentine and to a metal oxide layer will bond dental amalgam to
teeth.

123 . Dental amalgam:


A Needs to be 2 mm deep to have structural durability
B Is packed using an amalgam condenser
C Cannot be carved until fully set
D Avoids the need to check the occlusion
Well done, you have selected the right answer.
The correct answer is B

124. Composite resin:


A Should be placed under rubber dam
B Is more brittle than amalgam
C Does not require that unsupported enamel is removed
D Gives a perfect seal to a cavity
Answer: A. Rubber dam enhances moisture control, which is necessary for the bonding of the composite to
enamel/dentine.

125 . Bond strength of composite resins:


A Does not depend on the orientation of the enamel crystals to the bond interface
B Is not affected by the anisotropic nature of enamel
C To affected dentine can be 40% lower than to healthy dentine
D Is probably greater than published data suggests
Answer: C. Highly mineralised sclerotic dentine prevents plugs of resin from extending into the tubules.

126 . Incremental building of composite restorations is not necessary:


A Because there is no limit to the depth at which a light cure unit will cure a composite
B For restorations where the composite is less than 2–3 mm in depth
C If the composite is of considerable bulk
D Because it allows a greater amount of shrinkage to be accommodated by flow
Answer: B. Incremental placement does not alter the amount a composite resin shrinks, but allows a greater proportion
of the shrinkage to be accommodated by flow rather than bulk contraction.

127 . Creating contact points:


A Is easier with composite resin than amalgam
B Is made easier if there is significant shrinkage during setting
C Is enhanced by the packing procedure with amalgam
D Avoids the operator having to use wedges
Answer: C. Careful use of wedges and matrix bends is required for formation of contact points.

21 | P a g e
128 . Which one of the following statements is true?
A One of the advantages of resin composite inlays over ceramic inlays is that they can be used in larger cavities
B Resin composite inlays are more difficult to maintain than ceramic inlays
C CAD-CAM technology for the construction of inlays uses laser technology
D Resin composite inlays have a higher compressive strength than tensile strength
E Resin composite inlays resist wear and leakage
The correct answer is D

129 . Which one of the following statements is true?


A Ceramic and resin composite inlays have a similar coefficient of thermal expansion
B All inlays are contraindicated for bruxists
C In contrast to resin composite inlays, ceramic inlays are indicated for subgingival margin preparations
D CAD-CAM technology allows the omission of the laboratory construction of restorations
E Gold inlays have a greater tendency to wear than resin composite inlays
The correct answer is D
130 . Which one of the following statements is true?
A Surveyed crowns can be useful retainers for partial dentures
B Preparations for PFM crowns are more aggressive than all-ceramic crowns
C The main function of a temporary crown is to maintain dental appearance
D Porcelain-fused-to-metal crowns are indicated equally in young and old patients
E All-ceramic crowns are strong enough to be used in patients with occlusal parafunction
The correct answer is A

131 . Regarding porcelain-fused-to-metal crowns (PFMs), which one of the following statements is true?
A The junction of metal and ceramic should be absorbing most of the occlusal loads
B The metal should be absorbing most of the occlusal loads
C The ceramic should be absorbing most of the occlusal loads
D They should not be used in situations of high occlusal loads
E They are strong enough to withstand occlusal loads irrespective of which part of the crown absorbs the loads
The correct answer is B

132 . Regarding retention of the post crown, which one of the following statements is true?
A Post are used to reinforce and strengthen the root
B The ideal height of the ferrule is at least 1 mm
C An apical root filling of at least 2 mm should be maintained
D The length of the post should be one-third of the root length
E The axial wall of the ferrule must be at least 1 mm in thickness
The correct answer is E

133 . Regarding post crowns, which one of the following statements is true?
A Active posts are preferred to passive posts because they are more retentive
B Parallel-sided, serrated posts are the most retentive types of active posts
C Ideally, the post length should extend to 2 mm of the apex of the root
D A serrated post is less retentive than a threaded post
E Fibre posts are superior to cast posts
The correct answer is D

134 . Glass ionomer cements:


A Need to be 3 mm thick
B Simulate dentine and enamel equally well
C Do not ever require a conditioner
D Are inherently self-adhesive to enamel and dentine when moisture control is perfect

22 | P a g e
Answer: D. A conditioner (solution of the same polyalkenoic acid used by the manufacturer to form the cement) is
required only if the prepared tooth surface has been exposed to saliva and a pellicle formed.

135 . Glass ionomer cements:


A Are only chemically aired
B Can be desiccated by the use of rotating polishing discs
C Are tolerant of moisture contamination
D May be adjusted immediately after setting
The correct answer is B

136 . Provisional restorations:


A Should always be placed after a cavity is prepared
B Can be used to seal endodontic access cavities
C Are limited to zinc oxide/eugenol preparations
D Avoid the need to check the occlusion
The correct answer is B

137 . Which one of the responses below could be considered a disadvantage when discussing with the patient options
for a RRB in the upper central region?
A It is a reversible procedure and the RRB can be removed with minimal damage to the abutment tooth
B Patient preference due to cost, lack of tooth preparation and reduced chair time
C It is a technique-sensitive procedure and will require careful design and adequate isolation for cementation
D Conservative procedure and retention does not rely on conventional retention features
E Cost effective with studies suggesting a median survival time of up to 8 years
The correct answer is C

138 . Which one of the responses below is not a contraindication for RRB?
A Minimal enamel is present to bond to the RRB and a combination of enamel and dentine bonding is necessary
B Difficulty in isolation for cementation
C Excessive occlusal loading occurs and occlusal contacts are present on the pontics in excursive movements
D Heavily restored abutment teeth
E Translucent incisal edges of the abutment teeth
The correct answer is E

139 . Which one of the following statements is true?


A In a hybrid fixed–movable bridge the matrix component is usually incorporated within the adhesive bridge component
B All fixed movable bridges are contraindicated in long span situations
C When using conventional fixed–movable bridges, the pontic is connected rigidly to the smaller retainer at one end and
via a movable connector to the larger retainer at the other end
D Conventional fixed–fixed bridges are usually cemented in place using a resin cement
E In general, cantilever resin-retained bridges are not preferred over a fixed–fixed design because they are mechanically
weaker
The correct answer is B

140. Which one of the following statements is true?


A A pier abutment is a terminal abutment of a long span bridge
B A bridge retainer is the tooth to which the bridge is cemented
C With the advent of modern resin cements, resin-retained bridges can also be used confidently as long span bridges
D 2 retainers + 1 pontic + 2 connectors is equal to a 5-unit bridge
E Fixed–movable bridges can overcome the problem of divergent abutments
The correct answer is E

141 . Implants should ideally be placed:

23 | P a g e
A Immediately following tooth loss
B A year after tooth loss
C 8 weeks after tooth loss
D Within 4 weeks of tooth loss
The correct answer is C

142. A satisfactory clinical outcome with implants is determined by:


A Successful surgical placement of the implant
B Osseointegration of the implant
C Osseointegration of the implant and good chewing function
D Patient comfort and satisfaction with appearance
The correct answer is D

143. The most difficult challenge in achieving good appearance is encountered in a patient with:
A A low smile line
B A high smile line
C Good soft tissue biotype
D A high smile line with thin tissue biotype
The correct answer is D

144. Surgical placement of implants is guided by:


A A diagnostic wax up
B A surgical stent based on a diagnostic wax up
C A radiograph of the site
D A study model
The correct answer is B

145 . With regards to mandibular major connectors:


A A lingual bar is attached to the cingulae of the anterior teeth
B A sublingual bar requires at least 10.5 mm space between the floor of the mouth and the cingulae
C A labial bar is recommended in cases where the anterior teeth are proclined
D A labial bar would improve appearance
E A metallic anterior-tooth borne bar (Kennedy bar) is not indicated in cases of incisor spacing
Answer: E. A is false because it lies between 3 mm inferior to the gingival margins of the teeth. B is false because it
requires 10.5 mm minimally from the floor of the mouth to the gingival margins of the anterior teeth. C is false because
it is recommended when they are retroclined. D is false because the reverse is usually the case.

146 . With regards to partial denture retention:


A Indirect retainers are best used in tooth-borne cases
B Clasps are the most common type of retainer
C Clasps work by displacing the tooth
D Guide planes are mainly used for stability
E Precision attachments are rarely used nowadays
Answer: B. A is false because they are stabilising components where rotation is likely; they are therefore supporting
elements. C is false because they work by having to be distorted to pass over an undercut. D is false because they are
retaining devices, which may aid stability and appearance. E is false because they are commonly used in implant cases.

147 . Implant overdentures are a suitable treatment option for patients with:
A Denture retention problems
B Speech problems
C Difficulty chewing hard foods
D Cosmetic problems
The correct answer is A

24 | P a g e
148 . When planning implant surgery, what aids to diagnosis are routinely required?
A Diet history
B Primary casts only
C Primary casts and a panoramic radiograph
D Three-dimensional radiographic images (CT scan)
The correct answer is C

149. Precision attachments should:


A Be provided for all patients requiring a removable partial denture
B Be used for patients with denture retention problems
C Be used for patients as an aesthetic alternative to clasps
D Be used to provide better denture support
The correct answer is C

150. Teeth that are suitable for overdenture abutments should:


A Be adjacent to each other
B Be fully intact
C Be below the gingival margin
D Be symmetrically distributed and above the crestal bone level
The correct answer is D

151 . Precision attachments have:


A An unlimited lifespan
B No impact on periodontal attachment of natural teeth
C Significant requirement for interocclusal space
D No cost implications
The correct answer is C

152 . With regards to immediate insertion dentures (IIDs) which one of the following is true?
A IIDs are only supplied when teeth are extracted
B IIDs should be seen as a short-term treatment
C Residual ridge resorption should improve the fit of the IID
D IIDs should always have a full flange to enhance retention
E IIDs cannot be overdentures
Answer: B. A is false as they may follow extraction or decoronation. C is false as the reverse is the case, hence relining is
necessary. D is false because undercut ridges or patient preference may dictate that ridge lapping is employed. E is false
as they may be with either a retained root or an implant.

153 . With regards to the clinical stages of IIDs, which one of the following is true?
A Only one impression is required
B Stock trays never need to be customised to obtain an acceptable impression
C Non-perforated ‘special trays’ are best
D For add-ons, a registration and shade and mould do not need to be sent to the lab
E The patient should be informed at the onset about postextraction changes and the need for maintenance prior to the
definitive prosthesis being prescribed
Answer: E. A is false as impression of both arches should be recorded; only one may be required to make the IID on the
arch being considered. B is false because they often do need to be customised. C is false because perforations are very
much indicated to avoid the impression material being pulled away from the tray. D is false as it is always sensible to
send these to the technician.

154. With regards to retention of complete dentures, which one of the following is true?
A It is afforded directly by the denture-bearing tissues

25 | P a g e
B It is mainly dependant on the peripheral seal
C It is better when the saliva is viscous
D It may be improved by bony tuberosities
E It is tested by pushing the denture towards the tissues
Answer: B. A is false as it is mainly attributable to a peripheral seal. C is false as this may make it more problematic. D is
false because these may need to be relieved, which can result in a loss/ decrease in peripheral seal. E is false as
retention is resistance away from the tissues.

155. With regards to complete denture stability, which one of the following is true?
A This is the property of the denture that resists movement away from the tissues
B It is purely a matter of muscle balance
C Not all patients should be given balanced occlusion in RCP
D All patients must be given balanced articulation
E The selection of posterior teeth that are too large may result in instability
The correct answer is E

156 . What strength of ferric sulphate is used in deciduous teeth?


A 50%
B 5.5%
C 10.5%
D 15.5%
The correct answer is D

157. What is the medicament of choice for a vital pulpotomy?


A Ledermix
B Zinx oxide eugenol
C Ferric sulphate
D Formocresol
The correct answer is C

158. What is the name of the procedure used to restore a deciduous tooth when the pulp is NOT exposed?
A Direct pulp cap
B Indirect pulp cap
C Pulpectomy
D Pulpootomy
The correct answer is B

159. Which one of the following medicaments is NO LONGER used for deciduous teeth?
A Formocresol
B Ledermix
C Kalzinol
D Calcium hyodroxide
The correct answer is A

160 . Which one of the following is an indication for pulpectomy?


A Multiple teeth with deep caries and poor prognosis
B Unco-operative child
C Presence of an abscess
D Medically compromised child
The correct answer is C

161 . When considering pulpal treatment for traumatized teeth, which of the following signs is of LEAST significance?
A Extent of damage

26 | P a g e
B Negative response to vitality test at first visit after the trauma
C Stage of development of roots
D Possible contamination
E Time lapse since the trauma occurred
The correct answer is B

162. The ideal treatment for immature teeth with complicated large fractures is:
A Pulpectomy and RCT
B Pulpotomy and apexogenesis
C Pulpectomy and indirect pulp capping
D Direct pulp capping
E Immediate placement of a stainless steel crown
The correct answer is B

163 . The material recommended for direct capping of a small pulpal exposure resulting from a crown fracture is:
A Calcium hydroxide
B Glass ionomer
C Amalgam
D MTA
E Zinc oxide and eugenol
The correct answer is D

164. What is the most likely change in the pulp after a tooth is concussed or subluxated?
A Mineralisation
B Necrosis
C Internal resorption
D Irreversible pulpitis
E Reversible pulpitis
The correct answer is C

165. Which of the following is LEAST likely following avulsion of a mature anterior tooth?
A The return of vitality
B Root canal therapy
C Discoloration
D Root resorption
E Necrosis
The correct answer is A

166. Pulpectomy (complete pulp removal) is recommended when there is a diagnosis of:
A Reversible pulpitis
B Irreversible pulpitis
C Normal pulp
D Pulpitis in immature teeth
E Mineralised pulp
The correct answer is B

167. To which anatomical point should working length extend?


A Apical foramen
B Radiographic apex
C Apical constriction
D Enamel–dentinal junction
E The beginning of tertiary dentine deposition
The correct answer is C

27 | P a g e
168. Pulpotomy (partial removal of the pulp):
A Is never indicated
B Will relieve pain from a pulpitic tooth as readily as pulpectomy
C Should only be used as an emergency procedure for pain relief
D Should never be used for long-term treatment
E Should end 2–3 mm below the canal orifices
The correct answer is B

169. Access preparations in posterior teeth should:


A Be at least 5 mm deep
B Should be at the exact centre of the occlusal surface
C Include the mesial cusps
D Follow the outline of the pulp chamber
E When necessary include the marginal ridge
The correct answer is D

170. Coronal flaring:


A Serious weakens the tooth
B Can safely be achieved with hand files only
C Frequently leads to furcal perforations
D Makes cleaning and shaping the apical third of the canal easier
E Results in packing or ledging the canal
The correct answer is D

171. Which of the following would be the LEAST desirable property for a root canal filling material?
A Should not shrink
B Easy to place and remove
C Radiolucent
D Biocompatible
E Bacteriostatic
The correct answer is C

172. The percentage of pure gutta percha in gutta percha cones is:
A 80%
B 50%
C 10%
D 20%
E 5%
The correct answer is D

173. Thermoplasticised gutta-percha is:


A A hybrid material
B High melting point gutta percha
C Rigid and easy to place
D Used for lateral condensation techniques
E Identical in composition and properties to the gutta percha used in cones
The correct answer is B

174. The main component of sealers used for root canal obturation that is antibacterial and can be removed easily is:
A Resin
B Glass ionomer
C Bioceramics

28 | P a g e
D Zinc oxide/eugenol
E Silicone
The correct answer is D

175. The cement–dentine junction or apical constriction is:


A Present in teeth with apical resorption
B Present but thin in teeth with incomplete roots
C The ideal point at which a root filling should end
D Visible on periapical radiographs
E Easily located by tactile sensation
The correct answer is C

176. Which one of the following is NOT a primary objective of ‘voice control’?
A To gain the patient's attention
B To gain the patient's compliance
C To avert negative behaviour
D To establish authority
E To punish the child's improper behaviour
The correct answer is E

177. Which one of the following is NOT true of ‘reinforcement’?


A Strengthening of a pattern of behaviour, which increases the probability of that behaviour being displayed in the
future
B Verbal and non-verbal approval from dentist when the child responds positively to directions
C Should be given immediately and frequently
D Abrupt termination of treatment for a child patient with poor behaviour and return patient to parents
E None of the above
The correct answer is D

178. Which one of the following is NOT appropriate for promoting positive behaviour in a 3-year-old child?
A Parental absence in the surgery
B Modelling
C Tell show and do
D Reinforcement
E Desensitization
The correct answer is A

179. Local anaesthetics produce their anaesthetic effect by which one of the following?
A Blocking the brain's abilities to perceive pain
B Reducing the blood flow to the region being anaesthetised
C Selectively blocking the C-fibres responsible for nerve conduction
D Stabilising the neuronal membranes
E Effecting saltatory conduction of nerve impulses
The correct answer is D

180. The local anaesthetic articaine:


A Diffuses through bone better than lidocaine and mepivacaine
B Is mostly metabolised in the liver
C Has an increased half-life, which increases the risk of toxicity
D Is the anaesthetic of choice for inferior alveolar nerve blocks
E Can only be delivered using the Halstead method
The correct answer is A

29 | P a g e
181. For bupivacaine which one of these statements is true?
A Should be used in patients with liver dysfunction to avoid the use of lidocaine
B Is the anaesthetic of choice for children
C Of all local anaesthetic agents has the lowest risk of systemic toxicity
D Is the most potent of all amide anaesthetics
E Is pharmacologically similar to articaine
The correct answer is D

182. Which of the following types of conscious sedation does NOT require the use of pulse oximeter for monitoring of
oxygen saturation?
A Inhalation sedation
B Intravenous sedation
C Oral sedation
D Intramuscular sedation
E All of the above
The correct answer is A

183. Which one of the following is NOT true of ‘nitrous-oxygen sedation’?


A The level of oxygen should not exceed 50%
B Non-irritant to lung and nasal passage
C Minimal effect on cardiovascular function
D Minimal effect on respiratory function
E Use of local anaesthetic not required for painful procedure
The correct answer is E

184. To prevent diffusion hypoxia, the patient is recommended to:


A Breathe 100% oxygen for 5 minutes
B Breathe 100% oxygen for 15 minutes
C Gradually reduce the concentration of nitrous oxide and oxygen
D Breathe 90% oxygen for 5 minutes
E Breathe 90% oxygen for 15 minutes
The correct answer is A

185. In deciduous teeth, which one of the following is true?


A The carious process is different from that in permanent teeth
B Caries cannot be remineralised
C Caries rarely results in the need to extract teeth
D Caries always results in the formation of a perapical abscess
E Caries may result in the formation of an inter-radicular abscess
The correct answer is E

186. The possible consequences of leaving dental caries untreated do NOT include which one of the following?
A Pain
B Poor appetite
C Increased risk of dry socket after extraction
D Otitis media
E Brain abscess
The correct answer is C

187. When considering restoring children's teeth, which one of the following should NOT be taken into account?
A The child's attitudes to dental treatment
B The child's age
C The time until the tooth exfoliates

30 | P a g e
D The extent of the disease
E The availability of amalgam
The correct answer is E

188. Which one of the following is NOT true about the ‘Hall technique’?
A It is used in all carious primary molars
B No caries removal
C No tooth preparation is needed
D No local anaesthesia is necessary
E Hall crown cemented with glass ionomer luting cement
The correct answer is A

189. Which of the following adhesives has less predictable outcomes for dental bonding?
A Three-step etch-and-rinse
B Two-step etch-and-rinse
C Two-step self-etch
D One-step self-etch
E None of the above
The correct answer is D

190. Which of the following glass ionomer cements is used in the atraumatic restorative treatment?
A Conventional glass ionomer cement
B Resin-modified glass ionomer cement
C Metal-modified glass ionomer cement
D High-viscosity glass ionomer cement
E All of the above
The correct answer is D

191. Which one of the following statements is true?


A All children's first permanent molar teeth should be fissure sealed
B The cost-effectiveness of fissure sealants is independent of tooth morphology
C Glass ionomer sealants are best in situations where moisture control is poor
D The tooth must be prewashed and dried if contaminated by saliva after etching, before it can be sealed
The correct answer is C

192. What is the strength of orthophosphoric acid used to etch enamel?


A 10%
B 25%
C 37%
D 57%
The correct answer is C

193. Which one of these statements on NMES is true?


A NMES are not cariogenic
B Consumption should not exceed 60 g per day
C NMES should constitute no more than 15% of daily intake of sugars
D NMES are found in all sweet foods
The correct answer is B

194. Diet diaries should:


A Include week days only
B Only record what is eaten and drunk
C Only be used with adults

31 | P a g e
D Be given to all patients
E Be accompanied by written instructions
The correct answer is E

195. The most effective mechanism by which fluoride reduces caries susceptibility is:
A Effect on tooth morphology
B Incorporation into the structure of the developing tooth
C Effect on acid production by plaque
D Effect on chemical balance which promotes remineralisation
The correct answer is D

196. Which one of the following is NOT an effective public health measures for reducing caries level?
A Provision of fluoride toothpaste
B Fluoridation of public water supplies
C Widespread use of fluoride tablets and drops
D School fluoridated milk programmes
The correct answer is C

197. The partial displacement of a tooth out of its socket in an axial direction is a(n):
A Extrusion
B Intrusion
C Lateral luxation
D Avulsion
The correct answer is A

198. A complicated crown fracture is also known as:


A An enamel–dentine fracture
B A dentine–root fracture
C An enamel–root–pulp fracture
D An enamel–dentine–pulp fracture
The correct answer is D

199. A comminution injury is the:


A Crushing and compression of the tooth socket walls
B Fracture of the alveolus confined to the tooth socket walls
C Fracture of the alveolar process
D Fracture of the supporting bone
The correct answer is A

200. Which one of the following injuries does NOT require extraction of the traumatised deciduous tooth?
A Severe intrusion
B Complicated crown root fracture
C Palatally displaced incisor interfering with the occlusion
D Enamel fracture
The correct answer is D

201. Which one of the following is NOT a potential sequel to trauma to a deciduous tooth?
A Hypoplasia of the permanent successor
B Delayed eruption of the permanent successor
C Dilaceration of the permanent successor
D Macrodontia of the permanent successor
The correct answer is D

32 | P a g e
202. Which one of the following is an appropriate injury for repositioning and splinting deciduous teeth?
A Avulsion
B Severe intrusion
C Dentoalveolar fracture
D Concussion
The correct answer is C

203. Which one of the following is a suitable medicament for placement directly over exposed pulp tissue following
pulpectomy?
A MTA
B Amalgam
C Composite
D Glass ionomer
The correct answer is A

204. Repair of which one of the following injuries may NOT require local anaesthesia?
A Enamel fracture
B Enamel–dentine–pulp fracture
C Complicated crown–root fracture
D Crown–root fracture
The correct answer is A

205. Which one of the following is NOT an option for the management of a severe crown–root–pulp fracture?
A Repair with composite resin only
B Surgical/orthodontic extrusion
C Decoronation for bone preservation for implant in adulthood
D Extraction
The correct answer is A

206. Which one of the following describes a subluxation injury?


A Displacement of the tooth in an axial direction out of the socket
B No displacement or mobility but tender to percussion
C No displacement but mobility and tenderness to percussion
D Displacement of the tooth in a horizontal direction
The correct answer is C

207. For a tooth with a closed apex that has been intruded by 10 mm, which one of the following is the most
appropriate management strategy?
A Allow spontaneous re-eruption
B Orthodontic repositioning
C Surgical repositioning
D Extraction
The correct answer is C

208. An avulsed tooth with full root length but an open apex that has been out of its socket and kept dry for 90
minutes:
A Should not be replanted
B Will ankylose if replanted
C Can regain vitality if replanted
D Has a good prognosis for survival if replanted
The correct answer is B

209. The role of the dental team in child protection is to:

33 | P a g e
A Diagnose dental neglect
B Provide forensic reports for bite injuries
C Be alert to and report concerns of abuse
D Do all of the above
The correct answer is C

210. Which one of the following descriptions of injury would be most likely to raise suspicion of non-accidental injury in
children?
A Bruising in a baby or non-mobile child
B Multiple bruises of the shins on both legs in a 6-year-old child
C Abrasion to the forehead in a 2-year-old child
D An avulsed upper front tooth in an 8-year-old child
The correct answer is A

211. What is the MOST common use of the instrument shown in this picture?

A Elevation of surgical flap


B Protection of airway
C Retraction of cheek
D Retraction of a surgical flap
E Retraction of tongue
Answer: D. The instrument shown is a Bowdler Henry rake retractor, used for retraction of a surgical flap.

212. What is the MOST common use of the instrument shown in this picture?

A Elevation of surgical flap


B Protection of airway
C Retraction of cheek
D Retraction of surgical flap
E Retraction of tongue
Answer: C. The instrument shown is a Kilner cheek retractor.

213. The forceps shown in this picture are MOST appropriate for extraction of:

A Lower left first premolar


B Lower right first permanent molar
C Upper left first permanent molar

34 | P a g e
D Upper right first permanent molar
E Upper right first premolar
Answer: D. The forceps blades show a raised beak on the right blade and may be used to remove an upper right molar,
‘beaks to cheeks’.

214. The instrument in this picture is a:

A Cryer elevator
B Coupland elevator
C Warwick James elevator
D Luxator
E Cross bar elevator
Answer: A. The instrument shown has a triangular blade and is a left Cryer elevator.

215. What is the LEAST likely use of a Lack's tongue depressor during surgical removal of a lower third molar tooth?
A Protection of airway
B Protection of lingual nerve
C Protection of lingual soft tissues
D Retraction of lingual soft tissues
E Retraction of tongue
Answer: B. Protection of lingual nerve requires elevation of lingual soft tissues and placing an instrument between the
flap and the lingual cortex. This is accomplished by using a periosteal elevator and not with a Lack's tongue depressor.

216 . A 55-year-old female patient presents with a history of severe, sharp, shooting pain involving her left cheek. The
pain is spontaneous in origin and each episode lasts for approximately 30 seconds. The pain does not respond to routine
non-steroidal anti-inflammatory drugs. There are no remarkable findings on clinical and radiographic examination. What
is the most likely cause of her pain?
A Acute pulpitis
B Aggressive periodontitis
C Maxillary sinusitis
D Temporomandibular joint pain
E Trigeminal neuralgia
Answer: E. The features described are typical of trigeminal neuralgia. Odontogenic causes are unlikely due to the
absence of local disease and lack of response to NSAIDs

217. A 40-year-old male patient presents with a history of intense paroxysmal pain involving his right cheek for the last
few weeks. The pain is spontaneous in origin and often awakes him at midnight. He also reports similar episodes of pain
in the spring season last year. He also associates nasal stuffiness and facial flushing with pain episodes. There are no
other remarkable findings on clinical and radiographic examination. What is the most likely cause of his pain?
A Cluster headache
B Maxillary sinusitis
C Migraine
D Temporomandibular joint pain
E Trigeminal neuralgia
Answer: A. Paroxysmal pain in the mid-facial region with associated nasal stuffiness and facial flushing is indicative of
cluster headaches. It is more common in males and tends to occur at a specific time, hence the name ‘alarm clock
headache’.

35 | P a g e
218 . A 25-year-old female presents with a history of clicking and persistent tenderness involving her left
temporomandibular joint. She also reports recurrent ear aches and tinnitus. What is the most likely cause of her
symptoms?
A Internal derangement of TMJ
B Myofascial pain
C Osteoarthritis of TMJ
D Migraine
E Trigeminal neuralgia
Answer: A. Clicking of the TMJ and associated tenderness is typically associated with an internal derangement of TMJ.
Myofascial pain is more likely to be associated with tenderness in masticatory muscles. Osteoarthritis is unlikely at this
age.

219 . A 35-year-old patient presents with a history of recurrent meal time swelling and pain beneath his lower jaw. On
clinical examination you identify tenderness in the right submandibular gland and a 1 × 1 cm hard lump in the oral floor.
What is the most likely diagnosis?
A Ranula
B Sialoadenosis
C Sialolithiasis
D Sialometaplasia
E Sjögren syndrome
Answer: C. Recurrent meal time swelling of the submandibular gland may be associated with a stone in the
submandibular duct. The stone is usually palpable along the course of the submandibular duct. Sialosis is painless and
sialometaplasia is classically seen on the palate and presents with mucosal ulceration.

220 . A 45-year-old patient presents with a history of trigeminal neuralgia attends your practice. Which of the following
medications is LEAST likely to offer any benefit for this condition?
A Carbamazepine
B Chlorpromazine
C Clonazepam
D Gabapentin
E Phenytoin
Answer: B. Chlorpromazine is a typical antipsychotic most commonly used to treat schizophrenia.

221. You plan to carry out extraction of a lower left first molar, shown in this radiograph. Which one of the following
factors is MOST likely to increase the risk of fracture of the tooth during the extraction?

A Ankylosis of the mesial root


B Dilaceration of the distal root
C Presence of periapical radiolucency
D Previous endodontic treatment
E Relationship of the tooth to adjacent second molar
Answer: D. Prior endodontic treatment makes the tooth brittle and increases the risk of tooth fracture. There is no
evidence of ankylosis of the mesial root or dilacerations of the distal root.

222 . You have carried out a tooth extraction on a patient taking warfarin. Which one of the following is the most
appropriate additional measure to achieve haemostasis?

36 | P a g e
A Pack the extraction socket with Alveogyl and suture
B Pack the extraction socket with oxidised cellulose and suture
C Place a pressure pack over the extraction socket
D Prescribe acetylsalicylic acid postoperatively
E Prescribe tranexamic acid postoperatively
Answer: B. Tooth extraction in patients on oral anticoagulants merits additional measures to achieve haemostasis. The
most effective measure is to pack the socket with an artificial haemostatic agent (oxidised cellulose or gelfoam) and
place a horizontal mattress suture. Placement of a pressure pack is done routinely for all extractions and is not unique to
patients taking anticoagulants. Tranexamic acid prescription is usually not warranted in this case.

223. You are taking out a lower right second premolar tooth using extraction forceps. Which one of the following
movement is MOST appropriate to minimise the risk of crown fracture?
A Buccal displacement
B Lingual displacement
C Occlusal displacement
D Rotation
E Traction
Answer: D. Lower premolars are removed using apical pressure followed by rotation. Sudden movement in buccal or
lingual direction increases the risk of tooth fracture.

224. You plan to carry out extraction of the upper left first molar shown in this radiograph. Which one of the following is
the LEAST likely complication for this extraction?

A Creation of an oro-antral communication


B Dislodgement of residual amalgam restoration
C Fracture of the crown
D Tooth displacement into the antrum
E Tooth displacement into the nasal floor
Answer: E. Tooth displacement into the nasal floor is least likely because the anatomical location of the tooth is not
adjacent to the nasal cavity.

225 . You are supervising a junior colleague who is extracting an upper first premolar tooth using extraction forceps.
Which one of the following movement is MOST likely to cause root fracture during extraction?
A Apical pressure
B Buccal displacement
C Palatal displacement
D Rotation
E Traction
Answer: D. Upper first premolar usually has two roots and extraction involves apical pressure followed by progressive
bucco-palatal displacement; finally delivering the tooth in an bucco-occlusal direction. Rotation of this tooth is most
likely to result in fracture of one or both roots.

37 | P a g e
226 . Your patient requires surgical extraction of a retained root of a lower second premolar. You plan to raise a buccal
mucoperiosteal flap followed by bone removal. When obtaining an informed consent, which one of the following risks is
the MOST important consideration?
A Damage to buccal nerve
B Damage to incisive nerve
C Damage to inferior alveolar nerve
D Damage to lingual nerve
E Damage to mental nerve
Answer: E. The mental nerve is closely related to the roots of lower premolar teeth and is the one at greatest risk.

227 . You plan to carry out a surgical extraction of a fractured lower molar. Which one of the following methods is the
MOST appropriate for bone removal?
A A straight handpiece on a micromotor
B High-speed handpiece on air turbine
C Use of a bone file
D Use of mallet and chisel
E Use of Rongeur forceps
Answer: A. Bone removal during surgical extractions is accomplished using a straight surgical handpiece on a
micromotor with tungsten carbide burs. Use of air turbine can lead to surgical emphysema. Mallet and chisels can be
traumatic and not used commonly in contemporary practice.

228 . When using Coupland elevators, which one of the following precautions is LEAST likely to be applicable?
A Avoiding any force on adjacent teeth
B Avoiding tooth sectioning
C Protection of soft tissues
D Supporting the jaw bone
E Use of controlled force
Answer: B. Tooth sectioning is done frequently during surgical extractions and is not an impediment to the use of
Coupland elevators. In fact, tooth sectioning is meant to facilitate the extraction. All other precautions are mandatory.

229 . When gaining access to the operative site for surgical extractions, which one of the following factors is LEAST
relevant?
A Flap design
B Jaw symmetry
C Mouth opening
D Root morphology
E Tooth position
Answer: B. Jaw symmetry is unlikely to have any direct influence when planning access for surgical extractions

230. You are planning surgical removal of a fractured upper molar tooth. Which one of the following is the MOST
appropriate approach?
A Buccal mucoperiosteal flap
B Buccal mucosal flap
C Palatal mucoperiosteal flap
D Palatal mucosal flap
E None of the above
Answer: E. Access for surgical extractions is established using a full thickness (mucoperiosteal) flap through a buccal
approach.

231 . A 21-year-old patient presents to you with a partially erupted lower right third molar. When considering the
extraction of this tooth, which one of the following is the LEAST likely indication for extraction?
A Advanced caries
B Anterior crowding

38 | P a g e
C Recurrent pericoronitis
D Root resorption of adjacent tooth
E Severe pericoronitis
Answer: B. The presence of anterior crowding is not an indication for removal of wisdom teeth.

232 . You are examining an orthopantomogram (OPG) of a patient who requires extractions of his lower wisdom teeth.
Which one of the following findings indicates minimal risk of damage to the inferior alveolar nerve (IAN) during
extraction?
A Narrowing of IAN canal
B Diversion of IAN canal
C Darkening of root outline of impacted tooth
D Interrupting white lines of IAN canal
E Conical root outline of impacted tooth
Answer: E. The presence of concical root outline does not indicate an increased risk of damage to the inferior alveolar
nerve during extraction.

233 . When considering the treatment options for an impacted lower third molar, which one of the following factors is
MOST likely to influence your decision in favour of a coronectomy?
A Relationship of the impacted tooth to the adjacent tooth
B Relationship of the impacted tooth to the inferior alveolar nerve
C Relationship of the impacted tooth to the occlusal plane
D Relationship of the impacted tooth to the opposing tooth
E Relationship of the impacted tooth to the mandibular ramus
Answer: B. Increased risk of damage to inferior alveolar nerve, as assessed on radiographs, is the main reason to
consider coronectomy in favour of an extraction.

234 . You plan to carry out extraction of an impacted lower left third molar in your patient. With regards to the
angulation of the tooth, which one of the following is MOST likely to indicate a difficult extraction?
A Distoangular
B Horizontal
C Mesioangular
D Vertical
E None of the above
Answer: A. Distoangular impactions are generally considered to be the most challenging.

235 . A 25-year-old patient presents to you with recurrent pericoronitis related to the lower left third molar. After
assessment you advise surgical removal of this tooth under a local anaesthetic. Which one of the following
complications is LEAST likely to be discussed when obtaining an informed consent from the patient?
A Damage to buccal nerve
B Damage to inferior alveolar nerve
C Damage to lingual nerve
D Postoperative swelling
E Postoperative trismus
Answer: A. Risk of damage to long buccal nerve is not discussed routinely when obtaining an informed consent for
extractions of impacted third molars.

236 . An excisional biopsy is most appropriate for which one of the following lesions?
A Erythroplakia
B Fibroepithelial polyp
C Leukoplakia
D Melanoma
E Verrucous carcinoma

39 | P a g e
Answer: B. An excisional biopsy is appropriate for a benign lesion like a fibroepithelial polyp. Premalignant and
malignant disorders warrant an incisional biopsy in specialist settings.

237 . Diagnosis involving microscopic examination of living cells removed using a swab is termed:
A Autopsy
B Biopsy
C Culture and sensitivity
D Cytology
E None of the above
Answer: D. Specimens collected using swabs yield individual cells for microscopic examination. Biopsy involves
microscopic examination of tissue from a living subject while autopsy refers to microscopic examination of tissue
specimen from a dead subject.

238 . You are working as a general dental practitioner in primary care. A 58-year-old patient presents with a mass on his
tongue as shown in the figure. The patient is a smoker for the last 30 years. You suspected a squamous cell carcinoma.
Which one of the following is the most appropriate action for you?

A Perform an excisional biopsy


B Perform an incisional biopsy
C Prompt referral to an oral and maxillofacial surgeon
D Take a swab for cytology
E Urgent referral to an oral and maxillofacial surgeon
Answer: C. Suspicion of malignancy requires urgent referral to an oral and maxillofacial surgeon in the local secondary
care centre. Although confirmation of diagnosis involves an incisional biopsy, this is best carried out in specialist settings
who can provide definitive treatment and follow-up.

239. An incisional biopsy of oral and paraoral tissues may be performed to confirm the diagnosis of which one of the
following systemic conditions?
A Asthma
B Diabetes mellitus
C Peptic ulcers
D Rheumatoid arthritis
E Sjögren syndrome
Answer: E. Biopsy of the minor salivary glands of the lower lip may be undertaken to confirm the diagnosis of Sjögren
syndrome.

240 . When completing a biopsy form to accompany the specimen to the histopathology laboratory, which one of the
following is LEAST likely to aid the diagnosis?
A Intraoperative findings
B Relationship of the lesion to local tissues
C Site of the lesion
D Type of anaesthesia used
E Type of biopsy performed
Answer: D. Information on the type of anaesthesia used to perform a biopsy does not directly facilitate the pathologist
to establish a diagnosis.

40 | P a g e
241. You have carried out an extraction of the upper right first permanent molar tooth on your patient. You are
concerned about postoperative bleeding and intend to place a suture to aid haemostasis. Which one of the following is
the most appropriate suture in this situation?
A Buried intramucosal suture
B Continuous suture
C Horizontal mattress suture
D Simple interrupted suture
E Vertical mattress suture
Answer: C. A horizontal mattress suture is most appropriate to aid haemostasis following tooth extraction.

242 . You have carried out a surgical extraction of a lower second molar tooth and now you plan to close the flap. Which
one of the following would be the most appropriate suture for this purpose?
A 2-0 Vicryl
B 2-0 Silk
C 3-0 Vicryl
D 3-0 Nylon
E 3-0 Prolene
Well done, you have selected the right answer.
Answer: C. 3-0 Vicryl is the most appropriate suture.

243. Following a surgical extraction of a lower molar tooth you have closed the flap using a black silk suture. What is the
most appropriate time for suture removal?
A 1–2 Days
B 3–5 Days
C 5–7 Days
D 7–10 Days
E 10–14 Days
Answer: C. Non-resorbable sutures are usually removed in 5–7 days following the extraction.

244 . A 12-year-old child presents to you with a facial laceration on the right cheek area. Which one of the following
would be the most appropriate suture material for closure of the skin?
A Catgut
B Dexon
C Prolene
D Silk
E Vicryl
Answer: C. Skin closure is best carried out using Prolene.

245 . You plan to carry out surgical closure of an oroantral communication with a buccal sliding mucoperiosteal flap.
Which one of the following factors is most likely to increase the risk of wound dehiscence?
A Flap with bilateral releasing incisions
B Lack of postoperative antibiotics
C Tension across wound margins
D Use of local anaesthesia with a vasoconstrictor
E Use of non-resorbable suture material
Answer: C. Tension-free closure is the most important factor in preventing wound dehiscence.

246 . A 40-year-old male patient presents with severe pain related to a grossly decayed, unrestorable lower left first
molar. He has a poor oral hygiene and a neglected dentition. He smokes 20 cigarettes per day but is otherwise fit and
healthy. You carry out extraction of his LL6 under local anaesthesia using forceps alone. Which one of the following
complications is most likely?
A Alveolar osteitis
B Anaesthesia of the lingual nerve

41 | P a g e
C Haematoma of the oral floor
D Paraesthesia of the inferior alveolar nerve
E Postoperative bleeding
Answer: A. Smoking and poor oral hygiene are considered to be risk factors for the development of alveolar osteitis; it is
more likely with mandibular molar extractions. There is no specific risk for any of the other complications in this patient.

247 . A 70-year-old female patient requires extraction of her lower left second molar. She has a history of osteoporosis
and is taking oral alendronate. You are concerned about the risk of bisphosphonate-related osteonecrosis of the jaw.
Which one of the following concomitant factors is LEAST likely to increase the risk of this complication in patients taking
bisphosphonates?
A Diabetes mellitus
B Ischemic heart disease
C Poor oral hygiene
D Smoking
E Steroid therapy
Answer: B. Ischemic heart disease is unlikely to increase the risk of bisphosphonate-related osteonecrosis of jaws. All
other factors are relevant.

248 . A 60-year-old female patient requires extraction of her upper right first molar. She has a history of ischemic heart
disease and is taking oral warfarin. You are concerned about the risk of post-extraction bleeding. Which one of the
following is LEAST likely to aid haemostasis?
A Alveogyl
B Gelfoam
C Oxidised cellulose
D Pressure pack
E Suturing
Answer: A. Alveogyl is used as a dressing to manage alveolar osteitis and is not a haemostatic agent. All other agents and
measures will facilitate haemostasis.

249. A 45-year-old patient develops an oro-antral communication following extraction of her upper left first molar.
Following surgical repair of the defect, you provide postoperative advice to this patient. Which one of the following
precautions LEAST likely to be included in your instructions?
A Avoid forceful nose blowing
B Avoid smoking
C Avoid vigorous mouth rinsing
D Use a straw to drink liquids
E Use nasal decongestants
Answer: D It is not advisable to use a straw to drink liquids following repair of an oro-antral communication as it creates
a negative pressure in the mouth and may disturb the wound.

250 . Displacement of a tooth into the infratemporal space is most likely to be associated with which one of the
following teeth?
A Mandibular premolars
B Mandibular third molars
C Maxillary canines
D Maxillary premolars
E Maxillary third molars
Answer: E. Maxillary third molars are most likely to be displaced into the infratemporal space due to their anatomical
location.

251 . Which one of the following is the most appropriate indication for surgical endodontics?
A Draining sinus related to periradicular disease
B Inability to obdurate a canal due to ledge formation

42 | P a g e
C Instrument fracture in the root canal
D Lateral perforation during endodontics
E Non-resolving periradicular pathology
Answer: E. A persistent periradicular pathology that does not resolve with routine endodontics is the most common
reason to consider the option of surgical endodontics. Other issues can be managed with non-surgical endodontics in
most cases.

252 . When considering the option of surgical endodontics, which one of the following features is regarded as a
contraindication?
A Accessory root
B Bulbous roots
C Dilacerated root
D Extracoronal restorations
E Inadequate bone support
Answer: E. Lack of bone support will render the tooth unrestorable and an extraction may be warranted in most cases.

253 . Root-end resection (apicectomy) is performed routinely during surgical endodontics. Which one of the following is
the LEAST likely benefit of this step?
A Achieving haemostasis
B Elimination of lateral canals
C Facilitation of periradicular curettage
D Facilitation of retrograde root filling
E Improving access to periradicular tissues
Answer: A. Apicectomy does not directly facilitate haemostasis at the surgical site. All other benefits are correct.

254 . You plan to carry out a retrograde root filling following an apicectomy on an upper incisor. Which one of the
following materials is most likely be your first choice?
A Amalgam
B Composite
C Iodoform paste
D Mineral trioxide aggregate
E Zinc phosphate
Answer: D. Mineral trioxide aggregate is the material of choice for retrograde root fillings following an apicectomy.
Amalgam in no longer advisable.

255. Which one of the following measures is LEAST likely to be recommended following an apicectomy?
A Antibiotics
B Chlorhexidine mouth rinses
C Ice packs
D Non-steroidal anti-inflammatory agents
E Saline mouth rinses
Answer: A. In contemporary practice, antibiotics are not recommended following an apicectomy. All other measures are
beneficial.

256 . A 25-year-old pregnant subject in her first trimester presents with recurrent bleeding associated with a soft lump
on the facial gingival in the upper canine region. Your provisional diagnosis is a pyogenic granuloma. Which one of the
following is the most appropriate management option?
A Conservative management until parturition
B Prescribe antibiotics and review
C Refer to her gynaecologist
D Remove the lesion with a surgical scalpel
E Remove the lesion with an electrocautery

43 | P a g e
Answer: A. Pyogenic granulomas in pregnant subjects are best managed conservatively until parturition as spontaneous
resolution is possible thereafter. Removal during pregnancy is associated with a high recurrence rate.

257 . Which one of the following is the most common site for a traumatic neuroma?
A Lower premolar region
B Upper anterior region
C Upper lip
D Upper premolar region
E Ventral tongue
Answer: A. Lower premolar region (mental foramen) is the most common site for the occurrence of a traumatic
neuroma.

258 . Which one of the following lesions is most likely to recur after conservative surgical excision?
A Fibroma
B Lipoma
C Lymphangioma
D Neurofibroma
E Papilloma
Answer: C. Lymphangiomas may show an infiltrative growth pattern and are most likely to recur compared to other
lesions.

259. A pyogenic granuloma is most likely to be associated with which one of the following sites?

A Gingiva
B Hard palate
C Oral floor
D Tongue dorsum
E Ventral tongue
Answer: A. Although pyogenic granulomas may develop anywhere in the oral cavity, gingiva is the most common site.

260 . A 40-year-old female presents with a painless, sessile lump measuring 1 × 1 cm on her right buccal mucosa. The
lump is firm in consistency and the overlying mucosa is of normal colour. She has a history of chronic cheek biting.
Which one of the following is the most likely diagnosis?
A Epulis fissuratum
B Peripheral giant cell granuloma
C Pyogenic granuloma
D Traumatic fibroma
E Vascular malformation
Answer: D. The description best fits a fibroma developing secondary to trauma from cheek biting. Pyogenic granuloma
and peripheral giant cell granuloma are associated with bleeding and, along with vascular malformations, are coloured
lesions. Epulis fissuratum develops on the alveolar ridge as a result of denture irritation.

261 . A 50-year-old male presents with a swelling in his right posterior mandible. An OPG reveals a multilocular
radiolucency involving his right mandibular body and angle region. Which one of the following is the most likely
diagnosis?
A Adenomatoid odontogenic tumour
B Ameloblastoma
C Dentigerous cyst
D Odontoma
E Radicular cyst
Answer: B. Ameloblastoma characteristically presents as a multilocular radiolucent lesion in the posterior mandible.
Others are either unilocular radiolucencies or radiopaque lesions.

44 | P a g e
262 . A 45-year-old male presents with a swelling in his left posterior mandible. Radiographs show a large multilocular
radiolucency involving his left mandibular angle and ramus region. Following an incisional biopsy the lesion is confirmed
to be an ameloblastoma. The histopathology shows islands of enamel organ-type epithelium with reversed polarity and
evidence of a cystic change within the epithelium. Which one of the following histopathological variant of
ameloblastoma is this lesion most likely to represent?
A Acanthomatous
B Basal cell
C Desmoplastic
D Follicular
E Granular cell
Answer: D. Follicular type is the common histopathological variant of ameloblastoma and shows cystic changes.
Acanthomatous type shows squamous metaplasia, granular cell type shows eosinophilic granules while the desmoplastic
has a densely collagenised stroma. Basal cell type has cuboidal rather than columnar cells.

263. A 15-year-old female presents for a routine dental check-up. Radiographs reveal a 2 × 2 cm unilocular radiolucency
involving the crown of her unerupted canine. The radiolucency shows fine calcifications and extends apical to the
cement–enamel junction of the canine. Which one of the following is the most likely diagnosis?

A Adenomatoid odontogenic tumour


B Ameloblastoma
C Calcifying epithelial odontogenic tumour
D Dentigerous cyst
E Odontogenic keratocyst
Answer: A. Discovery of a radiolucency associated with an impacted maxillary canine in this age group may indicate a
dentigerous cyst. However, the presence of calcifications and apical extension of the lesion past the CEJ are more likely
to be associated with adenomatoid odontogenic tumour. Calcifying epithelial odontogenic tumour also shows
calcifications but tends to appear in mature adults and more commonly involves the posterior mandible.

264 . A 15-year-old female attends for a dental check-up with her mother who is concerned about delayed eruption of
her right maxillary canine. Radiographs reveal multiple radiopaque tooth-like structures associated with her unerupted
right maxillary canine. Which one of the following is the most likely diagnosis?
A Adenomatoid odontogenic tumour
B Complex odontoma
C Compound odontoma
D Dentigerous cyst
E Odontogenic keratocyst
Answer: C. Multiple tooth-like radiopacities are characteristic of a compound odontoma while a complex odontomes
appear as a single calcified mass.

265 . A 20-year-old male presents with a swelling in his right posterior mandible. An OPG reveals a radiopaque mass
attached to the mesial root of his lower right first permanent molar. Which one of the following is the most likely
diagnosis?
A Adenomatoid odontogenic tumor
B Amelobastic fibroma
C Ameloblastoma
D Cementoblastoma
E Odontogenic myxoma
Answer: D. This presentation is typical of a cementoblastoma. The other lesions are radiolucent.

266. A 14-year-old female patient presents with an unerupted right maxillary canine. Radiographs demonstrate a
unilocular radiolucency encompassing the crown of the impacted right maxillary canine. Which one of the following is
the most likely diagnosis?
A Dentigerous cyst

45 | P a g e
B Gingival cyst of the adult
C Lateral periodontal cyst
D Odontogenic keratocyst
E Radicular cyst
Answer: A. Pericoronal radiolucency associated with an impacted canine is most likely to represent a dentigerous cyst.

267. Which one of the following is the most common site for the occurrence of a keratocyst?
A Anterior mandible
B Anterior maxilla
C Maxillary antrum
D Posterior mandible
E Posterior maxilla
Answer: D. While odontogenic keratocysts can occur anywhere in the jaws and may involve the maxillary antrum later,
the posterior mandible is the most common site for their development.
268 . A 50-year-old male patient presents with a radiolucent lesion in his left posterior mandible. An incisional biopsy
confirms the diagnosis of an odontogenic keratocyst. Which one of the following is most important factor when planning
treatment for this lesion?
A High recurrence rate
B Intra operative bleeding
C Malignant transformation
D Risk of facial nerve damage
E Secondary infection
Answer: A. Odontogenic keratocysts have a thin friable lining and are notorious for a high recurrence rate.

269 . Which one of the following features is least likely to be associated with nevoid basal cell carcinoma (Gorlin–Coltz
syndrome)?
A Basal cell carcinomata
B Bifid ribs
C Frontal bossing
D Geographic tongue
E Keratocyts of mandible
Answer: D. Geographic tongue has no known association with this syndrome; other features are well-recognized
components.

270 . Osteosarcoma is considered to be a recognized complication of which one of the following conditions?
A Gardner syndrome
B McCune–Albright syndrome
C Osteoporosis
D Paget disease of bone
E Sjögren syndrome
Answer: D. Osteosarcoma is a recognised complication of Paget disease of bone. Gardner syndrome comprises benign
jaw osteomas with a risk of colorectal malignancy. Sjögren syndrome carries a risk of lymphoma. Other conditions are
not associated with a malignancy.

271. A 14-year-old girl presents with painless enlargement of her right maxilla. Radiographs demonstrate a poorly
demarcated ‘ground glass’ radiopacity involving her right maxilla. Which one of the following is the most likely
diagnosis?
A Fibrous dysplasia
B Cherubism
C Osteoporosis
D Paget disease of bone
E Ossifying fibroma
Answer: A. Ground glass opacity is characteristic of fibrous dysplasia. None of the other lesions have this presentation.

46 | P a g e
272 . A 12-year-old child is brought to you by his parents. They are concerned about progressive swelling involving both
sides of his lower jaw. You identify non-tender bilateral bone expansion involving the mandibular angle regions.
Radiographs reveal bilateral multilocular radiolucent defects involving the mandibular angle and ramus regions. Which
one of the following is the most likely diagnosis?
A Periapical cement-osseous dysplasia
B Cherubism
C Osteosarcoma
D Paget disease of bone
E Ossifying fibroma
Answer: B. Cherubism presents with bilateral mandibular involvement. Paget disease affects the elderly and mainly
involves the maxilla. Periapical cement-osseous dysplasia affects adults and involves the anterior mandible.
Osteosarcoma and ossifying fibroma are unilateral and tend to occur in older patients.

273 . Which one of the following conditions is most likely to be associated with multiple radiolucent defects in the
mandible?
A Acromegaly
B Addison disease
C Hyperparathyroidism
D Hyperthyroidism
E Phaeochromocytoma
Answer: C. Hyperparathyroidims is the only condition associated with well-defined lytic lesions in the mandible, often
referred to as ‘brown tumour’.

274 . Which one of the following is the most common primary bone malignancy affecting the jaws?
A Chondrosarcoma
B Ewing sarcoma
C Fibrosarcoma
D Multiple myeloma
E Osteosarcoma
Answer: E. Osteosarcoma is the commonest in jaws, followed by chondrosarcoma and Ewing sarcoma. Multiple
myeloma is a haematological malignancy (plasma cells), which may involve bone secondarily.

275 . A 35-year-old female patient presents with pain and clicking in her right temporomandibular joint for the last 6
months. Clinical examination does not reveal odontogenic cause. What does this condition most likely represent?
A Ankylosis of temporomandibular joint
B Fracture of the mandibular condyle
C Internal derangement of temporomandibular joint
D Myofascial pain
E Pericoronitis
Answer: C. Pain and clicking of TMJ is characteristic of an internal derangement.

276 . An 18-year-old female patient presents with tenderness and spasm of her masticatory muscles and difficulty in
opening her mouth. MRI scans of her temporomandibular joints are unremarkable. What would be your most likely
diagnosis?
A Ankylosis of temporomandibular joint
B Arthritis of temporomandibular joint
C Internal derangement of temporomandibular joint
D Myofascial pain
E Reiter syndrome
Answer: D. Pain localised to masticatory muscles and sparing the TMJ is characteristic of myofascial pain.

47 | P a g e
277 . An 8-year-child presents with severe limitation of his mouth opening. He has a history of trauma to his face after a
fall from a roof top 6 months ago. On clinical examination there is no clicking or tenderness in his temporomandibular
joints bilaterally. What is your most likely diagnosis?
A Ankylosis of temporomandibular joint
B Arthritis of temporomandibular joint
C Internal derangement of temporomandibular joint
D Myofascial pain
E Reiter syndrome
Answer: A. Marked limitation of mouth opening following childhood trauma may lead to ankylosis of the TMJ.

278 . A 45-year-old female patient is seen during a routine follow-up for severe clicking in her right temporomandibular
joint for the last 2 years. She has been diagnosed with an internal derangement of her right TMJ. She has tried
conservative measures but these have been largely unhelpful. Which one of the following may be the LEAST invasive
operative option to manage her condition?
A Arthrocentesis
B Condylectomy
C Coronoidectomy
D Diskectomy
E Eminectomy
Answer: A. Arthrocentesis has a documented success rate and is minimally invasive. Diskectomy may be a last resort for
patients with irreparable disk damage but is invasive. All other options are inappropriate.

279. A 35-year-old female banker presents with pain and clicking in her temporomandibular joints bilaterally. She also
has a history of bruxism. You make a provisional diagnosis of an internal derangement of her temporomandibular joints.
Which one of the following may be the LEAST appropriate measure to manage her condition?
A Prescription of analgesics
B Provision of a night guard
C Relaxation therapy
D Use of chewing gum
E Use of soft diet
Answer: D. Chewing gum is not advisable for this condition as it may cause fatigue to the TMJ and masticatory muscles.
All other measures are appropriate.

280 . A 19-year-old female patient presents with recurrent oral aphthous ulcers. Which one of the following conditions is
LEAST likely to be associated with this disorder?
A Folate deficiency
B Inflammatory bowel disease
C Iron deficiency
D Smoking
E Vitamin B12 deficiency
Answer: D. Smoking is associated with hyperkeratosis which may protect against aphthous ulcers.

281. A 16-year-old female patient presents with recurrent oral aphthous ulcers. You make a provisional diagnosis of
minor aphthous ulcers. Which one of the following features is LEAST likely to be associated with this condition?
A Healing within 10 days
B Lack of scarring
C Presence of vesicles
D Recurrences twice a year
E Size less than 1 cm
Answer: C. Minor apthous ulcers are not preceded by a vesicular phase. Vesiculation is a feature of herpes simplex virus
infection.
282 . A 28-year-old female patient presents with recurrent oral aphthous ulcers. You make a provisional diagnosis of
major aphthous ulcers. Which one of the following features is most likely to be associated with this condition?

48 | P a g e
A Healing within 10 days
B Lack of involvement of keratinised mucosa
C Lack of scarring
D Presence of vesicles
E Size more than 1 cm
Answer: E. Major aphthous ulcers are usually 1–3 cm in size. All other options are characteristic of minor apthous ulcers.

283 . A 60-year-old female patient presents with recurrent oral and corneal ulceration. Her biopsy specimen reveals
subepithelial bullae. What is your most likely diagnosis?
A Candidal infection
B Lichen planus
C Mucous membrane pemphigoid
D Pemphigus vulgaris
E Squamous cell carcinoma
Answer: C. Ocular involvement and subepithelial bullae are characteristic of benign mucous membrane pemphigoid.
Pemphigus vulgaris presents with intraepithelial vesiculation. Lichen planus does not show corneal involvement.
Candidal infection and oral squamous cell carcinoma do not present with subepithelial clefting.

284 . A 55-year-old male patient presents with haemorrhagic crusting of his lips following a course of antibiotic therapy.
He does not have any systemic manifestations. What would be your most likely diagnosis?
A Anaphylaxis
B Behçet syndrome
C Erythema multiforme
D Lichen planus
E Pemphigus vulgaris
Answer: C. The scenario is consistent with erythema multiforme. Anaphylaxis has systemic features. Haemorrhagic
crusting of lips is uncharacteristic of other disorders in the question.

285 . A 35-year-old female patient of South-Asian origin presents with a severe burning sensation in her mouth. She has
a history of betel quid chewing for the last 15 years. On clinical examination you note she has a marked trismus. Which
one of the following is the most likely diagnosis?
A Erythroplakia
B Leukoplakia
C Lichen planus
D Submucous fibrosis
E Tertiary syphilis
Answer: D. Betel quid use is strongly associated with oral submucous fibrosis but not with other lesions.

286 . A 60-year-old male patient presents with a white patch in his oral floor. He is a chronic smoker for the last 40
years. You make a provisional diagnosis of leukoplakia. What is the significance of leukoplakia at this site?
A Decreased risk of dysplasia
B Increased risk of acanthosis
C Increased risk of dysplasia
D Marked hyperkeratosis
E Marked lymphocytic infiltration
Answer: C. Leukoplakia of oral floor, ventral tongue and lip vermilion carries an increased risk of dysplasia, which is a key
determinant of the future course of treatment. Other options are less important.

287 . A 60-year-old female patient presents with a 3-week history of a painless ulcer on the left lateral border of her
tongue. She is a chronic cigarette smoker since her youth. Clinical examination shows the ulcer has reverted margins and
there is reduced mobility of her tongue. Which one of the following is the most likely diagnosis?

A Erythroplakia

49 | P a g e
B Leukoplakia
C Lichen planus
D Submucous fibrosis
E Squamous cell carcinoma
Answer: E. These scenario is consistent with the presentation of squamous cell carcinoma. Lesions associated with
leukoplakia and erythroplakia do not ulcerate until malignancy intervenes. Ulcerative lichen planus is painful and oral
submucous fibrosis is associated with a history of betel quid use.

288 . A 55-year-old patient presents with a red patch on the ventral aspect of his tongue. He is a chronic smoker. You
make a provisional diagnosis of erythroplakia. You plan to carry out an incisional biopsy of the lesion. Which one of the
following findings on histopathology is most likely to dictate the future course of treatment for this patient?
A Degree of acanthosis
B Degree of vascularity
C Presence of inflammatory cells
D Severity of dysplasia
E Severity of hyperkeratosis
Answer: D. Severity of dysplasia is the most important factor is determining the future course of treatment. All other
histopathological findings are minimally relevant.

289. You are discussing the premalignant potential of oral lesions with your colleagues. In this regard, which one of the
following is most likely to be associated with severe dysplasia on histopathological examination?
A Erosive lichen planus
B Erythroplakia
C Hyperplastic candidiasis
D Leukoplakia
E Syphilitic glossitis
Answer: B. Erythroplakia is most likely to present with dysplasia on microscopy compared to all other options.

290 . A 35-year-old male patient presents with recurrent meal time swelling in his right submandibular region. On clinical
examination you identify a hard lump in his oral floor. Which one of the following is the most likely diagnosis?
A Sialadenitis
B Sialolithiasis
C Sialosis
D Sialorrhoea
E Sialometaplasia
Answer: B. The scenario is consistent with a diagnosis of sialolithiasis. Sialosis often shows bilateral involvement of major
salivary glands. Sialorrhoea refers to increased salivary flow. Sialometaplasia typically affects the palate.

291 . A 18-year-old female patient presents with a soft, painless mucosal swelling on her right lower lip. The swelling has
been present for the last 3 months and shows recurrent change in size. It does not blanch on digital pressure. Which one
of the following is the most likely diagnosis?
A Fibroepithelial polyp
B Haemangioma
C Mucocele
D Pyogenic granuloma
E Ranula
Answer: C. Mucoceles are painless and show recurrent variation is size. A ranula typically presents in the oral floor. The
other lesions do not fluctuate in size.

292. A 18-year-female patient presents to you with bilateral, painless enlargement of her parotid glands. She has a
history of anorexia nervosa. Which one of the following is the most likely diagnosis?
A Sialadenitis
B Sialolithiasis

50 | P a g e
C Sialosis
D Sialorrhoea
E Sialometaplasia
Answer: C. Sialosis presents as a bilateral non-inflammatory, painless enlargement of parotid glands and may be
associated with eating disorders. Sialorrhoea refers to increased salivary flow. Sialadentis and sialolithiasis are unlikely
to involve parotid glands bilaterally.

293 . A 45-year-old patient presents with a 2 × 2 cm well-demarcated firm mass in his right parotid region. A fine-needle
aspiration cytology report shows evidence of a mixture of glandular epithelium and myoepithelial cells within a
mesenchymal background. What is the most likely diagnosis ?
A Acinic cell carcinoma
B Adenoid cystic carcinoma
C Adenolymphoma
D Pleomorphic adenoma
E Squamous cell carcinoma
Answer: D. The clinical and cytology findings are consistent with the diagnosis of a pleomorphic adenoma.
Adenolymphoma is composed of ductal epithelium and lymphoid stroma. There is no suggestion of a malignant lesion in
the scenario.

294 . A 45-year-old male patient presents with a painful lump in his right parotid region. On clinical examination there is
evidence of a mild weakness of his right facial nerve. Which one of the following is the most likely diagnosis ?
A Adenoid cystic carcinoma
B Adenolymphoma
C Basal cell adenoma
D Monomporphic adenoma
E Pleomorphic adenoma
Answer: A. Pain and facial nerve paralysis are typically associated with a malignant lesion of salivary glands. The
remaining options indicate benign lesions.

295. A 22-year-old male of Afro-Caribbean origin attends your practice with acute toothache. He has been taking
paracetamol and ibuprofen for pain relief. You note generalised pigmentation of his gingival mucosa. Which one of the
following is most likely to be responsible for this finding?
A Chronic inflammation
B Drug-related pigmentation
C Endocrine disturbance
D Melasma
E Racial pigmentation
Answer: E. Given the scenario, only racial pigmentation can be the correct answer. Ibuprofen and paracetamol do not
cause oral pigmentation. Melasma involves facial skin. There is no indication of an endocrine disturbance or mucosal
inflammation.

296. Which one of the following drugs is most likely to be associated with oral pigmentation?
A Amoxicillin
B Beclomethasone
C Codeine
D Propranolol
E Zidovudine
Answer: E. Zidovudine is an antiretroviral drug used for the management of HIV infection and is known to be associated
with oral mucosal pigmentation.

297. You are working as a general dental practitioner in primary care. A 28-years-old patient presents with perioral
pigmentation and reports a history of intestinal polyps. There are no other remarkable findings on clinical examination.
Which one of the following conditions is most likely to be associated with these features?

51 | P a g e
A Behçet syndrome
B Jaffe–Lichtenstein syndrome
C McCune–Albright syndrome
D Peutz–Jeghers syndrome
E Plummer–Vinson syndrome
Answer: D. Perioral pigmentation and intestinal polyps are characteristic of Peutz–Jeghers syndrome. Behçet syndrome
is associated ocular, oral and genital ulceration; Jaffe–Lichtenstein and McCune–Albright syndromes are associated with
polyostotic fibrous dysplasia; Plummer–Vinson syndrome is characterised by glossitis, dysphagia and oesophageal webs.

298. A 55-year-old male patient presents to you for a routine dental check-up. You note bilateral black pigmentation on
the buccal mucosa. He reports a recent diagnosis of an endocrine disturbance for which he has been prescribed 7.5 mg
prednisolone daily. Which one of the following systemic conditions is most likely to be associated with this case?
A Acromegaly
B Addison disease
C Hypothyroidism
D Myxoedema
E Phaeochromocytoma
Answer: B. Addison disease is associated with increased ACTH secretion, which may cause increased melanin
pigmentation of the buccal mucosa. The other options also represent endocrine disturbances but oral pigmentation is
not directly associated with any of these.

299 . A 45-year-old patient presents to your practice with an irregular black pigmented lesion on his buccal gingival
mucosa in the lower right posterior quadrant. You decide to take a periapical radiograph of the involved site. This
investigation is most likely to identify which one of the following?
A Amalgam tattoo
B HIV-related pigmentation
C Oral melanocytic nevus
D Peutz–Jeghers syndrome
E Smoker's melanosis
Answer: A. A periapical radiograph may only help identify radiopague amalgam deposits in the soft tissues. The other
conditions are limited to the mucosa and radiographs are not relevant.

300 . A 60-year-old patient presents to the surgery and informs you they have microcytic anaemia. Which of the
following laboratory values on a full blood count would be described as a microcytic anaemia?
A Low platelets
B Low white blood cell count
C Low haemoglobin and normal mean corpuscular volume
D Low haemoglobin and lowered mean corpuscular volume
E Low haemoglobin and raised mean corpuscular volume
Answer: D. The blood count would demonstrate a low haemoglobin and low mean corpuscular volume and on the blood
film the red cells would look microcytic.

301 . What is the most common cause of a vitamin B12 deficiency?


A Vegan diet
B Gastrointestinal malignancy
C Crohn's disease
D Ulcerative colitis
E Pernicious anaemia
Answer: E. Vitamin B12 deficiency can be caused by a vegan diet but pernicious anaemia is the most common cause.

302 . What is the normal life span of erythrocytes?


A 30 days
B 80 days

52 | P a g e
C 100 days
D 120 days
E 200 days
Answer: D. the average life span for a red blood cell is 120 days.

303 . Which of the following is NOT an orofacial sign of underlying anaemia?


A Recurrent aphthous stomatitis
B Angular cheilitis
C Glossitis
D Oral candidosis
E Oral lichen planus
Answer: E. Oral Lichen planus is a chronic autoimmune disorder and is not caused by anaemia. The other oral conditions
can be seen secondary to anaemia in a patient.

304 . The phagocytosis of bacteria by macrophages and neutrophils is referred to as what type of immune response?
A Adaptive response
B Hypersensitivity response
C Innate response
D Antibody response
E Angioedema
The correct answer is C

305 . Which antibodies are released in response to contact with an allergen in a hypersensitivity reaction?
A IgA
B IgG
C IgM
D IgE
E IgD
The correct answer is D

306 . Which of the following is the first-line treatment in a patient who has severe anaphylaxis in the dental surgery?
A Intramuscular adrenaline 1 : 1000 0.5 mL
B Intramuscular adrenaline 1 : 10 000 0.5 mL
C Intravenous adrenaline 1 : 10 000 0.5 mL
D Intravenous adrenaline 1 : 1000 0.5 mL
E Salbutamol nebuliser
The correct answer is A

307 . Which of the following is an example of an organ-specific autoimmune disease?


A Rheumatoid arthritis
B Sjögren syndrome
C Systemic lupus erythematous
D Scleroderma
E Type 1 diabetes mellitus
The correct answer is E

308 . What is the definition of hypertension?


A Blood pressure in excessive of 120/80 mmHg on 2 occasions
B Blood pressure in excessive of 140/90 mmHg on 3 occasions
C Blood pressure in excessive of 140/90 mmHg on 1 occasion
D Blood pressure in excessive of 160/100 mmHg on 1 occasion
E Blood pressure in excessive of 160/100 mmHg on 3 occasions
The correct answer is B

53 | P a g e
309 . Which of the following is NOT a side effect of the antihypertensive medication angiotensin converting enzyme
inhibitor (ACEI)?
A Dry mouth
B Glossitis
C Lichenoid reaction
D Angioedema
E Candidosis
The correct answer is E

310. Which of the following would be appropriate management for a patient having a myocardial infarction in dental
practice?
A Lie patient flat, give oxygen
B Phone 999, lie patient flat, give oxygen
C Phone 999, give GTN, 75 mg aspirin and oxygen
D Phone 999, give GTN, 150 mg aspirin and oxygen
E Phone 999, give GTN, 300 mg aspirin and oxygen
The correct answer is C

311 . A patient presents to the dental practice complaining of gingival swelling. They are on a number of medications.
Which of the following could be a cause of the gingival hyperplasia?
A Furosemide
B Atenolol
C Ramipril
D Losartan
E Nifedipine
The correct answer is E

312 . Which of the following is a common side effect in asthmatic patients using corticosteriod inhalers?
A Oral ulceration
B Pseudomembranous candidosis
C Lichenoid reaction
D Lichen planus
E Gingival hiperplasia
The correct answer is B

313 . A 60-year-old patient with known COPD becomes short of breath whilst you are carrying out treatment. Which of
the following would NOT be an appropriate management option?
A Sit them up
B Give oxygen
C Give a bronchodilator
D Lie them flat
E Give an inhaled steroid
The correct answer is D

314 . Which of the following respiratory conditions is NOT associated with a wheeze?
A Asthma
B Anaphylaxis
C COPD
D Allergy
E Sinusitis
The correct answer is E

54 | P a g e
314 . Which of the following gastroenterology disorders does NOT cause oral ulceration?
A Crohn's disease
B Ulcerative colitis
C Coeliac disease
D Irritable bowel syndrome
E Intrinsic factor deficiency
The correct answer is D

315 . A patient presents to the surgery with symptoms of bleeding and sore gums. You take a more detailed history from
the patient and it reveals they have a diet of tinned foods and no fresh fruit or vegetables. What deficiency might you
expect causing the gingival bleeding?
A Vitamin B deficiency
B Vitamin D deficiency
C Vitamin C deficiency
D Vitamin E deficiency
E Iron deficiency
The correct answer is C

316 . A patient presents to the surgery complaining of swelling of the lips and face. Which of the following conditions
would be the most likely underlying diagnosis?
A Ulcerative colitis
B Crohn's disease
C Irritable bowel syndrome
D Coeliac disease
E Gastroesophageal reflux disease
The correct answer is B

317 . Which of the following hormones is NOT secreted from the pituitary gland?
A Growth hormone (GH)
B Thyroid stimulating hormone (TSH)
C Luteinising hormone (LH)
D Triiodothyronine (T3)
E Follicle stimulating hormone (FSH)
The correct answer is D

318 . A patient presents to the dental surgery complaining of a burning sensation on the dorsum of the tongue. You
suspect burning mouth syndrome. Which of the following endocrine disorders would be most likely to cause this?
A Hyperthyroidism
B Diabetes mellitus
C Hypothyroidism
D Hypoparathyroidism
E Hyperparathyroidism
The correct answer is B

319 . A patient presents to the surgery and you notice features of weight gain in a central deposition, a moon face and
bruising on the arms and legs. Which of the following endocrine disorders would you suspect?
A Cushing syndrome
B Diabetes mellitus
C Acromegaly
D Addison disease
E Thyroid disease
The correct answer is A

55 | P a g e
320 . On a routine dental X-ray you notice reduced bone density, loss of lamina dura, root resorption and radiolucencies
in the mandible. Which of the following would be the most likely diagnosis?
A Diabetes mellitus
B Hyperthyroidism
C Hypothyroidism
D Hypoparathyroidism
E Hyperparathyroidism
The correct answer is E

321. What is the most prevalent renal disease in men?


A Urinary tract infection
B Acute renal failure
C Chronic renal failure
D Benign prostate hypertrophy
E Renal artery obstruction
The correct answer is D

322 . Which of the following statements is correct about managing patients in dental practice with chronic renal failure?
A Before medications are prescribed it is important to check the BNF for any dose adjustment required
B The patient should never have extractions in dental practice
C Non-steroidal anti-inflammatory drugs can be prescribed at a normal dose
D Dialysis patients do not need any special considerations in dental practice
E Patients are less likely to have diabetes mellitus
The correct answer is A

323 . A patient presents to dental practice with a known history of renal disease. They need a tooth extracted but you
are concerned about complications. Which of the following medications is likely to cause osteochemonecrosis?
A Corticosteroids
B Bisphosphonates
C Statin
D Aspirin
E Beta blocker
The correct answer is B

324 . On examining a patient you notice they have white flecks in their mouth consistent with oral candidosis. You want
to prescribe fluconazole tablets but which of the following medications would be a contraindication?
A Aspirin
B Corticosteroids
C Warfarin
D Clopidogrel
E Beta blockers
The correct answer is C

325 . Which of the following medications can cause lichenoid reactions?


A Aspirin
B Corticosteroids
C Warfarin
D Antihypertensives
E Clopidogrel
The correct answer is D

56 | P a g e
326 . Which of the following medications can cause dry mouth?
A Aspirin
B Corticosteriods
C Warfarin
D Amitriptylline
E Amoxicillin
The correct answer is D

327 . Which of the following are usually the first permanent teeth to erupt?
A Mandibular first premolars
B Mandibular first molars
C Maxillary central incisors
D Mandibular second molars
E Mandibular canines
The correct answer is B

328 . What is the incisor liability?


A Absence of the maxillary lateral incisors
B Difference in mesiodistal width of the primary and secondary incisors
C Increased overjet
D Reverse overjet
E Increased size of the permanent central incisors
The correct answer is B

329 . Which of the following is NOT a characteristic feature of the primary dentition?
A Spaced incisors
B Flush terminal molars
C Reverse overjet
D Reduced overbite
E Anthropoid spaces
The correct answer is C

330 . When assessing a patient's vertical skeletal pattern which clinical method can be used?
A Assessing where the soft tissue A and B points lie relative to zero meridian
B Assessing the Frankfort mandibular plane angle
C Calculation of the chin throat length
D Measurement of the upper lip length
The correct answer is B

331 . The LAFH is calculated as a percentage of the total face height (MFH + LAFH) and should be approximately?
A 55%
B 50%
C 65%
D 45%
E 60%
The correct answer is A

332 . Which one of the following is a Fox's plane used for?


A Measurement of the interpupillary distance
B To assess the presence of a maxillary cant
C To assess the presence of a mandibular cant
D To establish whether a mandibular asymmetry exists
The correct answer is B

57 | P a g e
333 . Which clinical indication would suggest a patient has undergone previous orthodontic treatment?
A The patient has generalised gingivitis
B The patient has a heavily restored dentition
C The patient has generalised decalcification on the labial surfaces of the teeth
D The patient suffers from minor apthous ulcers
The correct answer is C

334 . If a patient presents with an ectopic canine and on the periapical it reveals root resorption of the adjacent lateral
incisor, which one further radiological investigation would help diagnose the true extent of the root resorption?
A Upper standard occlusal
B Dental panoramic tomograph
C Lateral cephalogram
D Cone beam CT
The correct answer is D

335 . What type of appliance would you use to reduce an increased overbite?
A Removable appliance with a midline screw
B Removable appliance with posterior capping
C Aligners
D A Begg retainer
E Removable appliance with a flat anterior bite plane
The correct answer is E

336 . When using a functional appliance to treat a class II malocclusion, which dental change is not observed?
A Retroclination of the upper incisors
B Intrusion of the canines
C Proclination of the lower incisors
D Distal tipping of the maxillary dentition
E Mesial eruption of the mandibular buccal dentition
The correct answer is B

337 . What are Adams cribs that are used to retain a removable appliance on the first molars constructed from?
A 1 mm stainless steel
B 0.9 mm cobalt chromium
C 0.5 mm stainless steel
D 0.7 mm stainless steel
E 0.8 mm stainless steel
The correct answer is D

338 . In the initial stages of alignment with fixed appliances, what type of archwire would you select?
A Rectangular stainless steel archwire
B Round nickel titanium archwire
C Cobalt chromium archwire
D Beta-titanium archwire
E Round stainless steel archwire
The correct answer is B

339 . If you were to correct bilateral buccal crossbites, which appliance would you select to use in conjunction with fixed
appliance therapy?
A High-pull headgear

58 | P a g e
B Transpalatal arch
C Quadhelix
D A temporary anchorage device
E Nance arch
The correct answer is C

340 . Which feature is commonly seen in a patient with a thumb sucking habit?
A Retained deciduous incisors
B Incomplete overbite or localised anterior openbite
C Lateral openbites
D Retroclined upper incisors
The correct answers are B

341 . Which features would suggest orthodontic camouflage of a class II malocclusion would NOT be a feasible
treatment option in a patient that has stopped growing?
A Severely retrusive mandible
B Favourable soft tissue profile
C An ability to achieve lip competency following overjet reduction
D Mild crowding of lower arch
The correct answers are A

342 . Which one of the following presenting features is NOT favourable when considering use of a functional appliance
to correct a skeletal II discrepancy?
A Prepubertal patient
B Mandibular retrusion
C An average/reduced MMPA
D Uncontrolled epilepsy
E Increased overbite
The correct answer is D

343 . What sort of epidemiological study is used to investigate health determinants?


A Descriptive study
B Application study
C Population study
D Focussed study
E Analytical study
The correct answer is E

344 . What type of investigation are the series of Adult Dental Health Surveys that have been conducted approximately
once every decade since 1968?
A Screening studies
B Cross-sectional studies
C Cohort studies
D Clinical trials
E Case series
The correct answer is B

345 . According to the 2009 Adult Dental Health Survey, what percentage of the adult population of England had at least
one natural tooth in 2009?
A 100%
B 94%
C 89%

59 | P a g e
D 78%
E 72%
The correct answer is B

346 . According to the 2003 National Child Dental Health Survey, what percentage of children aged 5 years old had a
problem as a result of the condition of their teeth and gums within the preceding 12 months?
A 2%
B 12%
C 22%
D 32%
E 42%
The correct answer is C

347 . Which anatomical site of oral cancer has the best survival?
A Palate
B Gingiva
C Dorsum of tongue
D Lateral border of tongue
E Lip
The correct answer is E

348 . Which one of the following statements is true regarding inequalities in health in middle and high income
countries?
A Social inequalities in health are not a major problem in the wealthiest nations
B Social inequalities in health are not a major problem in the elderly
C The lower a person may be in the social hierarchy, the greater their risk of ill health
D There is only a gradient in poor health in socially disadvantaged groups
E Inequalities in health are a consequence of the equal distribution of health determinants
The correct answer is C

349 . Which one of the following statements is true regarding health education?
A It reduces health inequalities by having a greater effect on those most in need
B It reduces health inequalities by increasing the health of everyone to a higher level
C It reduces health inequalities by addressing the determinants of poor health
D It may increase health inequalities by being less effective for those in most need
E It may increase health inequalities by encouraging people to do something that they don't like, which may cause them
stress
The correct answer is D

350 . According to the research of Moles and Ashley, roughly what percentage of the total number of admissions of
children to hospital for the extraction of carious teeth came from children living in the most deprived 20% of
households?
A 10%
B 20%
C 30%
D 40%
E 50%
The correct answer is D

351 . What is the approximate 5-year survival rate for tongue cancer among those most affluent compared to those
most deprived?
A Affluent = three-quarters; deprived = one-half
B Affluent = three-quarters; deprived = one-third

60 | P a g e
C Affluent = one-half; deprived = one-third
D Affluent = one-half; deprived = one-quarter
E Affluent = one-third; deprived = one-quarter
The correct answer is C

352 . In the 2009 Adult Dental Health Survey, what percentage of people reported that they attended the dentist on a
regular basis?
A 51%
B 61%
C 71%
D 81%
E 91%
The correct answer is B

353 . Which of the following would NOT be an effective intervention to manage the anxiety of a patient with a low level
of anxiety?
A Modelling
B Environmental enhancement
C Rapport building
D Sending a gift such as a pen to the patient with their appointment letter
The correct answer is D

354 . Which of the following should NOT be used when seeking to manage the anxiety of a patient with a high level of
anxiety?
A Modelling
B Cognitive behaviour therapy
C Rapport building
D Talk to the patient about the causes of their anxiety
The correct answer is D

355 . In changing the environment in order to make it less anxiety provoking for patients, evidence suggests which one
of the following might be effective?
A The scent of lavender
B Restful pictures of fish
C Blue paint in the surgery
D Information on diet
The correct answer is A

356 . Which of the following constructs are not included in the Theory of Planned Behaviour?
A Intention
B Perceived behavioural control
C Actual behavioural control
D Risk perception
The correct answer is D

357 . The Implementation Intentions Model is NOT concerned with which one of the following?
A Statement of a plan about where the behaviour will happen
B Statement of when the behaviour will happen
C Statement of how the behaviour will happen
D Statement of why the behaviour will happen
The correct answer is D

358 . It is important to understand the models of functioning disability and health because:

61 | P a g e
A It enables clinicians to make decisions for our patients
B It enables us to understand Government policy
C It makes clinical care easier for the dental team
D It helps us to understand the effects of our actions, environment and systems on the lives of disabled people
The correct answer is D

359 . Good clinical management for patients requiring special care dentistry should include:
A Holding patients to make it easier to carry out treatment
B Making sure carers sign consent forms for adults unable to consent themselves
C Good history taking and risk assessment to make sure patients and staff are safe
D Refusing treatment for a patient if you don't have the facilities or expertise to treat them
The correct answer is C

360 . Good communication with adults requiring special care dentistry means:
A Raising your voice to patients so that they can hear you
B Finding out how the individual communicates normally and working with the patient and carers to ensure the best
possible experience
C Using texting or other IT methods for all communications
D Talking to accompanying persons to make sure they can relay the information to the patient
The correct answer is B

361 . A 43-year-old male patient asks for a privately paid for gold crown on his upper right lateral incisor. He wants to
have it done as within his culture it is symbolic of wealth and good fortune and it would, he says, elevate his social
standing. The patient is an irregular attender but his oral and dental health is good with no decay or restorations
present. In deciding whether or not to accede to his request which are the competing principles?
A Autonomy, beneficence and justice
B Autonomy, non-maleficence and justice
C Autonomy, non-maleficence and beneficence
D Non-maleficence, beneficence and justice
E Non-maleficence and justice
The correct answer is B

362 . According to the Hippocratic Oath which of the four principles might be regarded as a ‘guiding principle’?
A Autonomy
B Non-maleficence
C Beneficence
D Justice
E None
The correct answer is B

363 . Jimmy Streetwise is 11 years old. During half-term he attends his dentist, whom he has seen regularly since he was
3 years old, for an emergency appointment as a filling in LR6 has fallen out and is giving him pain. He is accompanied by
his grandfather who lives next door to Jimmy and is looking after him for the day. Jimmy's mother and father are at
work. Who can give consent for the examination and for the filling to be replaced (under LA) at that appointment?
A No-one, as only Jimmy's mother or father can give consent for treatment
B Only Jimmy
C The dentist provided his dental nurse agrees
D Jimmy or his grandfather
E Only the grandfather
Answer: D. Think whether or not the answer might change if (1) the tooth required extraction under general anaesthesia
or (2) Jimmy's grandfather lived abroad and only visited two or three times a year, and if so why?

62 | P a g e
364 . After attending his new dentist a few weeks for a course of treatment for replacement crowns and bridges, Mr
Watchful has noticed that the dentist would leave the room and come back in to carry on treatment without changing
his gloves and also often took instruments out of the sink to use on him. At the final appointment Mr Watchful
challenged the dentist on this, who just shrugged and said he had practised like this for years, as did many of his
colleagues in the area, and there had never been a problem. Which one of the following statements is correct?
A Mr Watchful would be entitled to a refund because his treatment had not been provided with reasonable skill and
care
B Mr Watchful would not succeed in claim for negligence because, if true, the dentist's infection control procedures
were being carried out in line with other local dentists
C Mr Watchful would be able to sue the dentist for inadequate infection control even though he had suffered no harm
D The dentist would be able to defend any claim against him, or any investigation by the GDC, because he was an
associate and not the practice owner
E The dentist would be at risk of an adverse finding by the Professional Conduct Committee in respect of his fitness to
practice because of inadequate infection control whether or not Mr Watchful suffered any ill-effects as a consequence
The correct answer is E

365 . The first step in risk management is:


A Assessment of the risk
B Develop a risk strategy
C Identify the risk
D Avoid the risk
E Minimise the risk
The correct answer is C

366 . Which of the following statements is the LEAST important in managing clinical risk?
A Communicating the risks and benefits to the patient
B Obtaining a signed consent form
C Working within your competency
D Maintaining good clinical records
E Having appropriate indemnity cover
The correct answer is B

367 . Which one of the following vaccinations is NOT currently recommended for dental staff?
A Hepatitis C
B Whooping cough
C TB
D Polio
E German measles
The correct answer is A

368 . Failure to adequately disinfect dental water lines can lead to contamination of the water supply with which one of
the following?
A Cyclosporiasis
B Schistosomiasis
C Legionella
D Cholera
E Chlostridium botulinum
The correct answer is C

369. When treating a known carrier of hepatitis C, which one of the following precautions should be taken:
A Ensure all staff have been immunized against hepatitis C
B Book appointment at the end of a session
C Only undertake non-invasive emergency treatment and refer

63 | P a g e
D Treat using standard precautions
E Dispose of all instruments at the end of treatment
The correct answer is D

370 . The primary role of the General Dental Council (GDC) is to:
A Oversee standards for undergraduate dental education in the UK
B Maintain a register of dental care professionals
C Discipline poorly performing dentists
D Protect patients
E Support Government in delivering the healthcare agenda
The correct answer is D

371 . Which one of the following statements is true?


A The GDC is a self-regulatory body
B The Chair of the GDC must be a dentist elected by the profession
C The GDC is the regulatory body for all dental care professionals in the UK
D The Care Quality Commission regulates NHS dental practices but has no jurisdiction over private practices
E A Health and Safety Executive Inspector must provide at least 48 hours notice prior to inspection of a dental practice
The correct answer is C

372 . A family who have been seen at your practice under the NHS have emigrated to New Zealand. What is the
MINIMUM period the clinical records have to be retained for the 37-year-old mother?
A 5 years
B 10 years
C 11 years
D 25 years
E Indefinitely
The correct answer is C

373. For the family in Question 1, what is the MINIMUM period the clinical records have to be retained for the 9-year-
old son?
A 9 years
B 11 years
C 12 years
D 16 years
E Indefinitely
The correct answer is D

374 . Information relating to a patient record can be released without the consent of the patient under which one of the
following circumstances?
A Request by General Dental Council
B Request by Court Order
C Request by Care Quality Commission
D Request by solicitor
E All of the above
The correct answer is B

375 . Which one of the following team roles was NOT identified by Belbin?
A The plant
B The shaper
C The creator
D The implementer
E The finisher

64 | P a g e
The correct answer is C

376 . Which one of the following is a mandatory core subject within the GDCs Continuing Professional Development?
A Legal and ethical issues
B Complaints handling
C Clinical record keeping
D Team working
E Disinfection and decontamination
The correct answer is E

377 . What two factors should you be clear about before communicating?
A That they need to know what I am saying and they'd better listen carefully
B What you want to convey and why you want to convey it
C Who has the fastest internet connection and are they on Twitter
D What time the practice closes and whether I can say it more quickly
The correct answer is B

378 . In terms of guidance for formulating the appropriate messages, what is essential?
A Networked computer system throughout the practice
B A personal assistant
C A practice business plan or set of practice objectives
D Direct access to a protection society adviser
The correct answer is C

379 . An agenda for a team meeting is:


A Not really necessary in a small environment such as a dental practice
B Useful for ensuring the meeting has objectives
C A tedious waste of everyone's time
D Best circulated electronically after the meeting has started
The correct answer is B

380 . If someone is standing in the practice facing you with their arms folded and their mouth shut:
A Are they likely to be very pleased to see you?
B Would you ‘copy’ their stance in order to communicate effectively?
C Can you assume that they are tense and possibly unhappy?
D Do you just walk away?
The correct answer is C

381 . Which of the following is NOT an external contact or customer?


A A dental supplier
B The Care Quality Commission
C A dental nurse
D The local newspaper
The correct answer is C

382. Which organisation's guidelines should be checked with regard to advertising standards?
A The British Dental Journal
B The General Dental Council
C The National Radiological Protection Board
D The British Billboard Association
The correct answer is B

383 . Writing prescriptions:

65 | P a g e
A Must always be handwritten
B Should be clear and also explained to the patient
C Can be scribbled as quickly as possible as the pharmacist can always ring to check the details
D All of the above
The correct answer is B

384 . Which one of the following need NOT be included in a referral letter?
A The practice address and contact details
B The patient's address and contact details
C The reason for referring the patient
D The dental school from which the referring dentist qualified
The correct answer is D

385 . On qualification a dentist, before being able to work, must register with which one of these organisations?
A The Care Quality Commission
B The British Dental Association
C The General Dental Council
D An insurance/indemnity company
The correct answer is C

386. A professional person is one who:


A Puts the interests of their patients/clients first
B Thinks their skills are the best
C Does not like life-long learning
D Does not think trust or integrity matter
The correct answer is A

66 | P a g e

You might also like